Joe Vincent Passmedicine O&G Flashcards

1
Q

You review a 34-year-old woman who is 13 weeks pregnant. During her previous pregnancy she developed pre-eclampsia and had to have a caesarean section at 36 weeks gestation. Her blood pressure both following the last pregnancy and today is normal. Which one of the following interventions should be offered to reduce the risk of developing pre-eclampsia again?

A. Prophylactic nifedipine therapy
B. Prophylactic labatelol therapy
C. Vitamin B6 supplementation
D. Extended folic acid supplementation
E. Low-dose aspirin

A

E. Low-dose aspirin

Hypertension in pregnancy

NICE published guidance in 2010 on the management of hypertension in pregnancy. They also made recommendations on reducing the risk of hypertensive disorders developing in the first place. Women who are at high risk of developing pre-eclampsia should take aspirin 75mg od from 12 weeks until the birth of the baby. High risk groups include:
hypertensive disease during previous pregnancies
chronic kidney disease
autoimmune disorders such as SLE or antiphospholipid syndrome
type 1 or 2 diabetes mellitus

The classification of hypertension in pregnancy is complicated and varies. Remember, in normal pregnancy:
blood pressure usually falls in the first trimester (particularly the diastolic), and continues to fall until 20-24 weeks. After this time the blood pressure usually increases to pre-pregnancy levels by term

Hypertension in pregnancy in usually defined as:
systolic > 140 mmHg or diastolic > 90 mmHg
or an increase above booking readings of > 30 mmHg systolic or > 15 mmHg diastolic

After establishing that the patient is hypertensive they should be categorised into one of the following groups:

Pre-existing hypertension
A history of hypertension before pregnancy or an elevated blood pressure > 140/90 mmHg before 20 weeks gestation

No proteinuria, no oedema

Occurs in 3-5% of pregnancies and is more common in older women

Pregnancy-induced hypertension
(PIH, also known as gestational hypertension)
Hypertension (as defined above) occurring in the second half of pregnancy (i.e. after 20 weeks)

No proteinuria, no oedema

Occurs in around 5-7% of pregnancies

Resolves following birth (typically after one month). Women with PIH are at increased risk of future pre-eclampsia or hypertension later in life

Pre-eclampsia
Pregnancy-induced hypertension in association with proteinuria (> 0.3g / 24 hours)

Oedema may occur but is now less commonly used as a criteria

Occurs in around 5% of pregnancies

How well did you know this?
1
Not at all
2
3
4
5
Perfectly
2
Q

A 62-year-old female presents as she feels she is becoming incontinent. She describes no dysuria or frequency, but commonly leaks urine when she coughs or laughs. What is the most appropriate initial management?

A. Bladder retraining
B. Topical oestrogen cream
C. Regular toileting
D. Trial of oxybutynin
E. Pelvic floor muscle training

A

E. Pelvic floor muscle training

Urinary incontinence - first-line treatment:
urge incontinence: bladder retraining
stress incontinence: pelvic floor muscle training

Urinary incontinence

Urinary incontinence (UI) is a common problem, affecting around 4-5% of the population. It is more common in elderly females.

Causes
overactive bladder (OAB)/urge incontinence: due to detrusor over activity
stress incontinence: leaking small amounts when coughing or laughing
mixed incontinence: both urge and stress
overflow incontinence: due to bladder outlet obstruction, e.g. due to prostate enlargement

Initial investigation
bladder diaries should be completed for a minimum of 3 days
vaginal examination to exclude cystocele
urine dipstick and culture

Management depends on whether urge or stress UI is the predominant picture. If urge incontinence is predominant:
bladder retraining (lasts for a minimum of 6 weeks, the idea is to gradually increase the intervals between voiding)
bladder stabilising drugs: antimuscarinic is first-line
surgical management: e.g. sacral nerve stimulation

If stress incontinence is predominant:
pelvic floor muscle training (for a minimum of 3 months)
surgical procedures: e.g. retropubic mid-urethral tape procedures

How well did you know this?
1
Not at all
2
3
4
5
Perfectly
3
Q

A 31-year-old woman presents as she has noted an offensive, fishy vaginal discharge. She describes a grey, watery discharge. What is the most likely diagnosis?

A. Trichomonas vaginalis
B. Candida
C. Chlamydia
D. Bacterial vaginosis
E. Physiological discharge

A

D. Bacterial Vaginosis

Vaginal discharge

Vaginal discharge is a common presenting symptom and is not always pathological

Common causes:
physiological
Candida
Trichomonas vaginalis
bacterial vaginosis

Less common causes:
whilst cervical infections such as Chlamydia and Gonorrhoea can cause a vaginal discharge this is rarely the presenting symptoms
ectropion
foreign body
cervical cancer

Key features of the common causes are listed below

Candida = ‘Cottage cheese’ discharge, Vulvitis, Itch
Trichomonas vaginalis = Offensive, yellow/green, frothy discharge, Vulvovaginitis, Strawberry cervix
Bacterial vaginosis = Offensive, thin, white/grey, ‘fishy’ discharge

How well did you know this?
1
Not at all
2
3
4
5
Perfectly
4
Q

Which one of the following is less common in women who take the combined oral contraceptive pill?

A. Stroke
B. Endometrial cancer
C. Pulmonary embolism
D. Cervical cancer
E. Ischaemic heart disease

A

B. Endometrial cancer

Combined oral contraceptive pill
increased risk of breast and cervical cancer
protective against ovarian and endometrial cancer

Combined oral contraceptive pill: advantages/disadvantages

Advantages of combined oral contraceptive pill
highly effective (failure rate

How well did you know this?
1
Not at all
2
3
4
5
Perfectly
5
Q

Theme: Pelvic pain

A. Appendicitis
B. Ovarian torsion
C. Urogenital prolapse
D. Endometriosis
E. Urinary tract infection
F. Ovarian cyst
G. Threatened miscarriage
H. Irritable bowel syndrome
I. Pelvic inflammatory disease
J. Ectopic pregnancy

For each one of the following scenarios please select the most likely diagnosis:

A 24-year-old woman presents with mild, crampy suprapubic pain and light vaginal bleeding. Her last period was 10 weeks ago. Vaginal examination shows a small amount of blood around the cervix but is otherwise unremarkable.

A

G. Threatened miscarriage - The question asks what is the most likely diagnosis. The mild suprapubic pain at 10 weeks gestation is more characteristic of a miscarriage than an ectopic pregnancy. In clinical practice this patient would be referred the same day for an ultrasound scan.

Pelvic pain

In women the most common cause of pelvic pain is primary dysmenorrhoea. Some women also experience transient pain in the middle of their cycle secondary to ovulation (mittelschmerz). The table below gives characteristic features for other conditions causing pelvic pain:

Usually acute

Ectopic pregnancy - A typical history is a female with a history of 6-8 weeks amenorrhoea who presents with lower abdominal pain and later develops vaginal bleeding
Shoulder tip pain and cervical excitation may be seen

Urinary tract infection - Dysuria and frequency are common but women may experience suprapubic burning secondary to cystitis

Appendicitis - Pain initially in the central abdomen before localising to the right iliac fossa
Anorexia is common
Tachycardia, low-grade pyrexia, tenderness in RIF
Rovsing’s sign: more pain in RIF than LIF when palpating LIF

Pelvic inflammatory disease - Pelvic pain, fever, deep dyspareunia, vaginal discharge, dysuria and menstrual irregularities may occur
Cervical excitation may be found on examination

Ovarian torsion - Usually sudden onset unilateral lower abdominal pain. Onset may coincide with exercise.
Nausea and vomiting are common
Unilateral, tender adnexal mass on examination

Miscarriage - Vaginal bleeding and crampy lower abdominal pain following a period of amenorrhoea

Usually chronic

Endometriosis - Chronic pelvic pain
Dysmenorrhoea, pain often starts days before bleeding
Deep dyspareunia
Subfertility

Irritable bowel syndrome - Extremely common. The most consistent features are abdominal pain, bloating and change in bowel habit
Features such as lethargy, nausea, backache and bladder symptoms may also be present

Ovarian cyst -Unilateral dull ache which may be intermittent or only occur during intercourse. Torsion or rupture may lead to severe abdominal pain
Large cysts may cause abdominal swelling or pressure effects on the bladder

Urogenital prolapse - Seen in older women
Sensation of pressure, heaviness, ‘bearing-down’
Urinary symptoms: incontinence, frequency, urgency

How well did you know this?
1
Not at all
2
3
4
5
Perfectly
6
Q

Theme: Pelvic pain

A. Appendicitis
B. Ovarian torsion
C. Urogenital prolapse
D. Endometriosis
E. Urinary tract infection
F. Ovarian cyst
G. Threatened miscarriage
H. Irritable bowel syndrome
I. Pelvic inflammatory disease
J. Ectopic pregnancy

For each one of the following scenarios please select the most likely diagnosis:

A 67-year-old woman presents with a heavy, dragging sensation in the suprapubic region. She also has frequency and urgency.

A

C. Urogenital prolapse

Women who have a urogenital prolapse typically describe a ‘bearing down’, ‘heaviness’ or ‘dragging’ sensation.Women who have a urogenital prolapse typically describe a ‘bearing down’, ‘heaviness’ or ‘dragging’ sensation.

Pelvic pain

In women the most common cause of pelvic pain is primary dysmenorrhoea. Some women also experience transient pain in the middle of their cycle secondary to ovulation (mittelschmerz). The table below gives characteristic features for other conditions causing pelvic pain:

Usually acute

Ectopic pregnancy - A typical history is a female with a history of 6-8 weeks amenorrhoea who presents with lower abdominal pain and later develops vaginal bleeding
Shoulder tip pain and cervical excitation may be seen

Urinary tract infection - Dysuria and frequency are common but women may experience suprapubic burning secondary to cystitis

Appendicitis - Pain initially in the central abdomen before localising to the right iliac fossa
Anorexia is common
Tachycardia, low-grade pyrexia, tenderness in RIF
Rovsing’s sign: more pain in RIF than LIF when palpating LIF

Pelvic inflammatory disease - Pelvic pain, fever, deep dyspareunia, vaginal discharge, dysuria and menstrual irregularities may occur
Cervical excitation may be found on examination

Ovarian torsion - Usually sudden onset unilateral lower abdominal pain. Onset may coincide with exercise.
Nausea and vomiting are common
Unilateral, tender adnexal mass on examination

Miscarriage - Vaginal bleeding and crampy lower abdominal pain following a period of amenorrhoea

Usually chronic

Endometriosis - Chronic pelvic pain
Dysmenorrhoea, pain often starts days before bleeding
Deep dyspareunia
Subfertility

Irritable bowel syndrome - Extremely common. The most consistent features are abdominal pain, bloating and change in bowel habit
Features such as lethargy, nausea, backache and bladder symptoms may also be present

Ovarian cyst -Unilateral dull ache which may be intermittent or only occur during intercourse. Torsion or rupture may lead to severe abdominal pain
Large cysts may cause abdominal swelling or pressure effects on the bladder

Urogenital prolapse - Seen in older women
Sensation of pressure, heaviness, ‘bearing-down’
Urinary symptoms: incontinence, frequency, urgency

How well did you know this?
1
Not at all
2
3
4
5
Perfectly
7
Q

Theme: Pelvic pain

A. Appendicitis
B. Ovarian torsion
C. Urogenital prolapse
D. Endometriosis
E. Urinary tract infection
F. Ovarian cyst
G. Threatened miscarriage
H. Irritable bowel syndrome
I. Pelvic inflammatory disease
J. Ectopic pregnancy

For each one of the following scenarios please select the most likely diagnosis:

A 29-year-old woman presents with suprapubic pain, irregular periods, dysuria and pain during intercourse. There is cervical excitation on examination.

A

I. Pelvic inflammatory disease

Cervical excitation is found in both pelvic inflammatory disease and ectopic pregnancy.

Pelvic pain

In women the most common cause of pelvic pain is primary dysmenorrhoea. Some women also experience transient pain in the middle of their cycle secondary to ovulation (mittelschmerz). The table below gives characteristic features for other conditions causing pelvic pain:

Usually acute

Ectopic pregnancy - A typical history is a female with a history of 6-8 weeks amenorrhoea who presents with lower abdominal pain and later develops vaginal bleeding
Shoulder tip pain and cervical excitation may be seen

Urinary tract infection - Dysuria and frequency are common but women may experience suprapubic burning secondary to cystitis

Appendicitis - Pain initially in the central abdomen before localising to the right iliac fossa
Anorexia is common
Tachycardia, low-grade pyrexia, tenderness in RIF
Rovsing’s sign: more pain in RIF than LIF when palpating LIF

Pelvic inflammatory disease - Pelvic pain, fever, deep dyspareunia, vaginal discharge, dysuria and menstrual irregularities may occur
Cervical excitation may be found on examination

Ovarian torsion - Usually sudden onset unilateral lower abdominal pain. Onset may coincide with exercise.
Nausea and vomiting are common
Unilateral, tender adnexal mass on examination

Miscarriage - Vaginal bleeding and crampy lower abdominal pain following a period of amenorrhoea

Usually chronic

Endometriosis - Chronic pelvic pain
Dysmenorrhoea, pain often starts days before bleeding
Deep dyspareunia
Subfertility

Irritable bowel syndrome - Extremely common. The most consistent features are abdominal pain, bloating and change in bowel habit
Features such as lethargy, nausea, backache and bladder symptoms may also be present

Ovarian cyst -Unilateral dull ache which may be intermittent or only occur during intercourse. Torsion or rupture may lead to severe abdominal pain
Large cysts may cause abdominal swelling or pressure effects on the bladder

Urogenital prolapse - Seen in older women
Sensation of pressure, heaviness, ‘bearing-down’
Urinary symptoms: incontinence, frequency, urgency

How well did you know this?
1
Not at all
2
3
4
5
Perfectly
8
Q

You review a 28-year-old woman who is 26 weeks pregnant. She has just had a routine glucose tolerance test as her BMI is 34 kg/m^2. The following results were obtained:

Time (hours) Blood glucose (mmol/l)
0 7.1
2 11.2

There have been no other antenatal problems and her anomaly scan was normal. What is the most appropriate action?

A. Start metformin + advice about diet / exercise
B. Start metformin + advice about diet / exercise + self-monitor glucose levels
C. Advice about diet / exercise + self-monitor glucose levels
D. Start insulin + advice about diet / exercise + self-monitor glucose levels
E. Advise weight loss + start metformin

A

C. Advice about diet/exercise + self-monitorins glcose levels

Most women with gestational diabetes can be managed with a combination of diet and self-monitoring.

Pregnancy: diabetes mellitus

Diabetes mellitus may be a pre-existing problem or develop during pregnancy, gestational diabetes. It complicates around 1 in 40 pregnancies

Risk factors for gestational diabetes
BMI of > 30 kg/m^2
previous macrosomic baby weighing 4.5 kg or above.
previous gestational diabetes
first-degree relative with diabetes
family origin with a high prevalence of diabetes (South Asian, black Caribbean and Middle Eastern)

Screening for gestational diabetes
if a women has had gestational diabetes previously an oral glucose tolerance test (OGTT) should be performed at 16-18 weeks and at 28 weeks if the first test is normal
women with any of the other risk factors should be offered an OGTT at 24-28 weeks
currently the same WHO diagnostic criteria are used as for non-pregnant patients. There is however increasing evidence that a lower threshold should be used as treating borderline patients improves both maternal and neonatal outcomes

NICE issued guidelines on the management of diabetes mellitus in pregnancy which were updated in 2008

Management of pre-existing diabetes:
weight loss for women with BMI of > 27 kg/m^2
stop oral hypoglycaemic agents, apart from metformin, and commence insulin
folic acid 5 mg/day from pre-conception to 12 weeks gestation
detailed anomaly scan at 18-20 weeks including four-chamber view of the heart and outflow tracts
tight glycaemic control reduces complication rates
treat retinopathy as can worsen during pregnancy

Management of gestational diabetes
responds to changes in diet and exercise in around 80% of women
oral hypoglycaemic agents (metformin or glibenclamide) or insulin injections are needed if blood glucose control is poor or this is any evidence of complications (e.g. macrosomia)
there is increasing evidence that oral hypoglycaemic agents are both safe and give similar outcomes to insulin
hypoglycaemic medication should be stopped following delivery
a fasting glucose should be checked at the 6 week postnatal check

How well did you know this?
1
Not at all
2
3
4
5
Perfectly
9
Q

A 22-year-old woman who is an immigrant from Malawi presents for review as she thinks she is pregnant. This is confirmed with a positive pregnancy test. She is known to be HIV positive. Which one of the following should NOT be part of the management plan to ensure an optimal outcome?

A. Oral zidovudine for the newborn until 6 weeks of age
B. Maternal antiretroviral therapy
C. Encourage breast feeding
D. Intrapartum zidovudine infusion
E. Elective caesarean section

A

C. Encourage breast feeding

The 2008 BHIVA guidelines suggest vaginal delivery may be an option for women on HAART who have an undetectable viral load but whether this will translate into clinical practice remains to be seen

HIV and pregnancy

With the increased incidence of HIV infection amongst the heterosexual population there are an increasing number of HIV positive women giving birth in the UK. In London the incidence may be as high as 0.4% of pregnant women. The aim of treating HIV positive women during pregnancy is to minimise harm to both the mother and fetus, and to reduce the chance of vertical transmission.

Guidelines regularly change on this subject and most recent guidelines can be found using the links provided.

Factors which reduce vertical transmission (from 25-30% to 2%)
maternal antiretroviral therapy
mode of delivery (caesarean section)
neonatal antiretroviral therapy
infant feeding (bottle feeding)

Screening
NICE guidelines recommend offering HIV screening to all pregnant women

Antiretroviral therapy
all pregnant women should be offered antiretroviral therapy regardless of whether they were taking it previously
if women are not currently taking antiretroviral therapy the RCOG recommend that it is commenced between 28 and 32 weeks of gestation and should be continued intrapartum. BHIVA recommend that antiretroviral therapy may be started at an earlier gestation depending upon the individual situation

Mode of delivery
vaginal delivery is recommenced if viral load is less than 50 copies/ml at 36 weeks, otherwise caesarian section is recommended
a zidovudine infusion should be started four hours before beginning the caesarean section

Neonatal antiretroviral therapy
zidovudine is usually administered orally to the neonate if maternal viral load is

How well did you know this?
1
Not at all
2
3
4
5
Perfectly
10
Q

A 24-year-old woman who is 18 weeks pregnant presents for review Earlier on in the morning she came into contact with a child who has chickenpox. She is unsure if she had the condition herself as a child. What is the most appropriate action?

A. Advise her to present within 24 hours of the rash developing for consideration of IV aciclovir
B. Reassure her that there is no risk of fetal complications at this point in pregnancy
C. Give varicella immunoglobulin
D. Check varicella antibodies
E. Prescribe oral aciclovir

A

D. Check varicella antibodies - Chickenpox exposure in pregnancy - first step is to check antibodies

If there is any doubt about the mother previously having chickenpox maternal blood should be checked for varicella antibodies

Chickenpox exposure in pregnancy

Chickenpox is caused by primary infection with varicella zoster virus. Shingles is reactivation of dormant virus in dorsal root ganglion. In pregnancy there is a risk to both the mother and also the fetus, a syndrome now termed fetal varicella syndrome

Fetal varicella syndrome (FVS)
risk of FVS following maternal varicella exposure is around 1% if occurs before 20 weeks gestation
studies have shown a very small number of cases occurring between 20-28 weeks gestation and none following 28 weeks
features of FVS include skin scarring, eye defects (microphthalmia), limb hypoplasia, microcephaly and learning disabilities

Management of chickenpox exposure
if there is any doubt about the mother previously having chickenpox maternal blood should be checked for varicella antibodies
if the pregnant women is not immune to varicella she should be given varicella zoster immunoglobulin (VZIG) as soon as possible. RCOG and Greenbook guidelines suggest VZIG is effective up to 10 days post exposure
consensus guidelines suggest oral aciclovir should be given if pregnant women with chickenpox present within 24 hours of onset of the rash

How well did you know this?
1
Not at all
2
3
4
5
Perfectly
11
Q

A 36-year-old woman presents for a routine antenatal review. She is now 15 weeks pregnant. Her blood pressure in clinic is 154/94 mmHg. This is confirmed with ambulatory blood pressure monitoring. On reviewing the notes it appears her blood pressure four weeks ago was 146/88 mmHg. A urine dipstick is normal. There is no significant past medical history of note. What is the most likely diagnosis?

A. Pre-eclampsia
B. Pregnancy-induced hypertension
C. White-coat hypertension
D. Normal physiological change
E. Pre-existing hypertension

A

E. Pre-existing hypertension

This lady has pre-existing hypertension. Pregnancy related blood pressure problems (such as pregnancy-induced hypertension or pre-eclampsia) do not occur before 20 weeks. The raised ambulatory blood pressure readings exclude a diagnosis of white-coat hypertension.

Note the use of the term ‘pre-existing hypertension’ rather than essential hypertension. Raised blood pressure in a 36-year-old female is not that common and raises the possibility of secondary hypertension.

Hypertension in pregnancy

NICE published guidance in 2010 on the management of hypertension in pregnancy. They also made recommendations on reducing the risk of hypertensive disorders developing in the first place. Women who are at high risk of developing pre-eclampsia should take aspirin 75mg od from 12 weeks until the birth of the baby. High risk groups include:
hypertensive disease during previous pregnancies
chronic kidney disease
autoimmune disorders such as SLE or antiphospholipid syndrome
type 1 or 2 diabetes mellitus

The classification of hypertension in pregnancy is complicated and varies. Remember, in normal pregnancy:
blood pressure usually falls in the first trimester (particularly the diastolic), and continues to fall until 20-24 weeks
after this time the blood pressure usually increases to pre-pregnancy levels by term

Hypertension in pregnancy in usually defined as:
systolic > 140 mmHg or diastolic > 90 mmHg
or an increase above booking readings of > 30 mmHg systolic or > 15 mmHg diastolic

After establishing that the patient is hypertensive they should be categorised into one of the following groups:

Pre-existing hypertension
A history of hypertension before pregnancy or an elevated blood pressure > 140/90 mmHg before 20 weeks gestation

No proteinuria, no oedema

Occurs in 3-5% of pregnancies and is more common in older women

Pregnancy-induced hypertension
(PIH, also known as gestational hypertension)
Hypertension (as defined above) occurring in the second half of pregnancy (i.e. after 20 weeks)

No proteinuria, no oedema

Occurs in around 5-7% of pregnancies

Resolves following birth (typically after one month). Women with PIH are at increased risk of future pre-eclampsia or hypertension later in life

Pre-eclampsia
Pregnancy-induced hypertension in association with proteinuria (> 0.3g / 24 hours)

Oedema may occur but is now less commonly used as a criteria

Occurs in around 5% of pregnancies

How well did you know this?
1
Not at all
2
3
4
5
Perfectly
12
Q

Theme: Ovarian cysts

A. Dermoid cyst (teratoma)
B. Endometriotic cyst
C. Granulosa cell tumour
D. Clear cell tumour
E. Corpus luteum cyst
F. Mucinous cystadenoma
G. Follicular cyst
H. Serous cystadenoma
I. Dysgerminoma
J. Fibroma

For each one of the following please select the answer from the list above:

Most common type of ovarian pathology associated with Meigs’ syndrome

A

J. Fibroma

Ovarian cysts: types

Benign ovarian cysts are extremely common. They may be divided into physiological cysts, benign germ cell tumours, benign epithelial tumours and benign sex cord stromal tumours

Physiological cysts (functional cysts)

Follicular cysts
commonest type of ovarian cyst
due to non-rupture of the dominant follicle or failure of atresia in a non-dominant follicle
commonly regress after several menstrual cycles

Corpus luteum cyst
during the menstrual cycle if pregnancy doesn’t occur the corpus luteum usually breaks down and disappears. If this doesn’t occur the corpus luteum may fill with blood or fluid and form a corpus luteal cyst
more likely to present with intraperitoneal bleeding than follicular cysts

Benign germ cell tumours

Dermoid cyst
also called mature cystic teratomas. Usually lined with epithelial tissue and hence may contain skin appendages, hair and teeth
most common benign ovarian tumour in woman under the age of 30 years
median age of diagnosis is 30 years old
bilateral in 10-20%
usually asymptomatic. Torsion is more likely than with other ovarian tumours

Benign epithelial tumours

Arise from the ovarian surface epithelium

Serous cystadenoma
the most common benign epithelial tumour which bears a resemblance to the most common type of ovarian cancer (serous carcinoma)
bilateral in around 20%

Mucinous cystadenoma
second most common benign epithelial tumour
they are typically large and may become massive
if ruptures may cause pseudomyxoma peritonei

How well did you know this?
1
Not at all
2
3
4
5
Perfectly
13
Q

Theme: Ovarian cysts

A. Dermoid cyst (teratoma)
B. Endometriotic cyst
C. Granulosa cell tumour
D. Clear cell tumour
E. Corpus luteum cyst
F. Mucinous cystadenoma
G. Follicular cyst
H. Serous cystadenoma
I. Dysgerminoma
J. Fibroma

For each one of the following please select the answer from the list above:

Most common benign ovarian tumour in women under the age of 25 years

A

Theme: Ovarian cysts

A. Dermoid cyst (teratoma)
B. Endometriotic cyst
C. Granulosa cell tumour
D. Clear cell tumour
E. Corpus luteum cyst
F. Mucinous cystadenoma
G. Follicular cyst
H. Serous cystadenoma
I. Dysgerminoma
J. Fibroma

For each one of the following please select the answer from the list above:

Most common type of ovarian pathology associated with Meigs’ syndromeOvarian cysts: types

Benign ovarian cysts are extremely common. They may be divided into physiological cysts, benign germ cell tumours, benign epithelial tumours and benign sex cord stromal tumours

Physiological cysts (functional cysts)

Follicular cysts
commonest type of ovarian cyst
due to non-rupture of the dominant follicle or failure of atresia in a non-dominant follicle
commonly regress after several menstrual cycles

Corpus luteum cyst
during the menstrual cycle if pregnancy doesn’t occur the corpus luteum usually breaks down and disappears. If this doesn’t occur the corpus luteum may fill with blood or fluid and form a corpus luteal cyst
more likely to present with intraperitoneal bleeding than follicular cysts

Benign germ cell tumours

Dermoid cyst
also called mature cystic teratomas. Usually lined with epithelial tissue and hence may contain skin appendages, hair and teeth
most common benign ovarian tumour in woman under the age of 30 years
median age of diagnosis is 30 years old
bilateral in 10-20%
usually asymptomatic. Torsion is more likely than with other ovarian tumours

Benign epithelial tumours

Arise from the ovarian surface epithelium

Serous cystadenoma
the most common benign epithelial tumour which bears a resemblance to the most common type of ovarian cancer (serous carcinoma)
bilateral in around 20%

Mucinous cystadenoma
second most common benign epithelial tumour
they are typically large and may become massive
if ruptures may cause pseudomyxoma peritonei

How well did you know this?
1
Not at all
2
3
4
5
Perfectly
14
Q

Theme: Ovarian cysts

A. Dermoid cyst (teratoma)
B. Endometriotic cyst
C. Granulosa cell tumour
D. Clear cell tumour
E. Corpus luteum cyst
F. Mucinous cystadenoma
G. Follicular cyst
H. Serous cystadenoma
I. Dysgerminoma
J. Fibroma

For each one of the following please select the answer from the list above:

The most common cause of ovarian enlargement in women of a reproductive age

A

Ovarian cysts: types

Benign ovarian cysts are extremely common. They may be divided into physiological cysts, benign germ cell tumours, benign epithelial tumours and benign sex cord stromal tumours

Physiological cysts (functional cysts)

Follicular cysts
commonest type of ovarian cyst
due to non-rupture of the dominant follicle or failure of atresia in a non-dominant follicle
commonly regress after several menstrual cycles

Corpus luteum cyst
during the menstrual cycle if pregnancy doesn’t occur the corpus luteum usually breaks down and disappears. If this doesn’t occur the corpus luteum may fill with blood or fluid and form a corpus luteal cyst
more likely to present with intraperitoneal bleeding than follicular cysts

Benign germ cell tumours

Dermoid cyst
also called mature cystic teratomas. Usually lined with epithelial tissue and hence may contain skin appendages, hair and teeth
most common benign ovarian tumour in woman under the age of 30 years
median age of diagnosis is 30 years old
bilateral in 10-20%
usually asymptomatic. Torsion is more likely than with other ovarian tumours

Benign epithelial tumours

Arise from the ovarian surface epithelium

Serous cystadenoma
the most common benign epithelial tumour which bears a resemblance to the most common type of ovarian cancer (serous carcinoma)
bilateral in around 20%

Mucinous cystadenoma
second most common benign epithelial tumour
they are typically large and may become massive
if ruptures may cause pseudomyxoma peritonei

How well did you know this?
1
Not at all
2
3
4
5
Perfectly
15
Q

A 31-year-old woman presents for review. For the past few months she has been feeling generally tired and has not had a normal period for around 4 months. Prior to this she had a regular 30 day cycle. A pregnancy test is negative, pelvic examination is normal and routine bloods are ordered:

FBC Normal
U&E Normal
TFT Normal
Follicle-stimulating hormone 41 iu/l ( 100 pmol/l)

What is the most likely diagnosis?

Ovarian cancer
Gonadotropin-producing pituitary adenoma
Turner syndrome
Premature ovarian failure
Aromatase enzyme deficiency
A

Premature ovarian failure

Premature ovarian failure is defined as the onset of menopausal symptoms and elevated gonadotrophin levels before the age of 40 years.

Causes
idiopathic - the most common cause
chemotherapy
autoimmune
radiation

Features are similar to those of the normal climacteric but the actual presenting problem may differ
climacteric symptoms: hot flushes, night sweats
infertility
secondary amenorrhoea
raised FSH, LH levels

How well did you know this?
1
Not at all
2
3
4
5
Perfectly
16
Q

A 33-year-old female presents with a vaginal discharge. Which one of the following features is not consistent with bacterial vaginosis?

Vaginal pH > 4.5
Thin, white homogenous discharge
Strawberry cervix
Clue cells on microscopy
Positive whiff test
A

A strawberry cervix is associated with Trichomonas vaginalis, a condition which may present in a similar fashion to bacterial vaginosis

Bacterial vaginosis

Bacterial vaginosis (BV) describes an overgrowth of predominately anaerobic organisms such as Gardnerella vaginalis. This leads to a consequent fall in lactic acid producing aerobic lactobacilli resulting in a raised vaginal pH.

Whilst BV is not a sexually transmitted infection it is seen almost exclusively in sexually active women.

Features
vaginal discharge: ‘fishy’, offensive
asymptomatic in 50%

Amsel’s criteria for diagnosis of BV - 3 of the following 4 points should be present
thin, white homogenous discharge
clue cells on microscopy: stippled vaginal epithelial cells
vaginal pH > 4.5
positive whiff test (addition of potassium hydroxide results in fishy odour)

Management
oral metronidazole for 5-7 days
70-80% initial cure rate
relapse rate > 50% within 3 months
the BNF suggests topical metronidazole or topical clindamycin as alternatives

© Image used on license from PathoPic
Clue cells - epithelial cells develop a stippled appearance due to being covered with bacteria

Bacterial vaginosis in pregnancy
results in an increased risk of preterm labour, low birth weight and chorioamnionitis, late miscarriage
it was previously taught that oral metronidazole should be avoided in the first trimester and topical clindamycin used instead. Recent guidelines however recommend that oral metronidazole is used throughout pregnancy. The BNF still advises against the use of high dose metronidazole regimes

How well did you know this?
1
Not at all
2
3
4
5
Perfectly
17
Q

Which one of the following statements regarding endometrial cancer is incorrect?

Trans-vaginal ultrasound is the first-line investigation
Has a poor prognosis
Progestogen treatment may be used in frail elderly patients not fit for surgery
Treatment of early disease is with total abdominal hysterectomy with bilateral salpingo-oophorectomy
Pelvic pain is rarely a presenting feature
A

Endometrial cancer

Endometrial cancer is classically seen in post-menopausal women but around 25% of cases occur before the menopause. It usually carries a good prognosis due to early detection

The risk factors for endometrial cancer are as follows*:
obesity
nulliparity
early menarche
late menopause
unopposed oestrogen. The addition of a progestogen to oestrogen reduces this risk (e.g. In HRT). The BNF states that the additional risk is eliminated if a progestogen is given continuously
diabetes mellitus
tamoxifen
polycystic ovarian syndrome

Features
post-menopausal bleeding is the classic symptom
pre-menopausal women may have a change intermenstrual bleeding
pain and discharge are unusual features

Investigation
first-line investigation is trans-vaginal ultrasound - a normal endometrial thickness (

How well did you know this?
1
Not at all
2
3
4
5
Perfectly
18
Q

A 29-year-old female presents to her GP as she missed her Micronor pill (progestogen only) this morning and is unsure what to do. She normally takes the pill at around 0830 and it is now 1100. What advice should be given?

Take missed pill now and no further action needed
Emergency contraception should be offered
Take missed pill now and advise condom use until pill taking re-established for 48 hours
Take missed pill now and omit pill break at end of pack
Perform a pregnancy test
A

Progestogen only pill: missed pill

The missed pill rules for the progestogen only pill (POP) are simpler than those used for the combined oral contraceptive pill, but it is important not to confuse the two.

‘Traditional’ POPs (Micronor, Noriday, Nogeston, Femulen)
If less than 3 hours late
no action required, continue as normal

If more than 3 hours late (i.e. more than 27 hours since the last pill was taken)
action needed - see below

Cerazette (desogestrel)
If less than 12 hours late
no action required, continue as normal

If more than 12 hours late (i.e. more than 36 hours since the last pill was taken)
action needed - see below

Action required, if needed:
take the missed pill as soon as possible. If more than one pill has been missed just take one pill. Take the next pill at the usual time, which may mean taking two pills in one day
continue with rest of pack
extra precautions (e.g. condoms) should be used until pill taking has been re-established for 48 hours

How well did you know this?
1
Not at all
2
3
4
5
Perfectly
19
Q

A woman who is 34 weeks pregnant is found to have an amniotic fluid volume of 440 ml. Which one of the following conditions is not part of the differential diagnosis?

Premature rupture of membranes
Pre-eclampsia
Tracheo-oesophageal fistula
Renal agenesis
Intrauterine growth restriction
A

An amniotic fluid volume of 440ml indicates oligohydramnios. Tracheo-oesophageal fistula is associated with polyhydramnios.

Oligohydramnios

In oligohydramnios there is reduced amniotic fluid. Definitions vary but include less than 500ml at 32-36 weeks and an amniotic fluid index (AFI)

How well did you know this?
1
Not at all
2
3
4
5
Perfectly
20
Q

A 25-year-old woman presents for her first cervical smear. What is the most important aetiological factor causing cervical cancer?

Human papilloma virus 6 & 11
Early first intercourse
Smoking
Combined oral contraceptive pill use
Human papilloma virus 16 & 18
A

Whilst all of the above are known to contribute to the development of cervical cancer infection with human papilloma virus 16 & 18 is by far the most important factor.

Cervical cancer

The incidence of cervical cancer peaks around the 6th decade. It may be divided into
squamous cell cancer (80%)
adenocarcinoma (20%)

Features
may be detected during routine cervical cancer screening
abnormal vaginal bleeding: postcoital, intermenstrual or postmenopausal bleeding
vaginal discharge

Risk factors
human papilloma virus 16,18 & 33
smoking
human immunodeficiency virus
early first intercourse, many sexual partners
high parity
lower socioeconomic status
combined oral contraceptive pill*

How well did you know this?
1
Not at all
2
3
4
5
Perfectly
21
Q

Which one of the following clinical features would be least consistent with a diagnosis of severe pre-eclampsia?

Headache
Epigastric pain
Reflexes difficult to elicit
Low platelet count
Papilloedema
A

Severe pre-eclampsia is associated with hyperreflexia and clonus. A low platelet count may indicate the patient is developing HELLP syndrome

Pre-eclampsia

Pre-eclampsia is a condition seen after 20 weeks gestation characterised by pregnancy-induced hypertension in association with proteinuria (> 0.3g / 24 hours). Oedema used to be third element of the classic triad but is now often not included in the definition as it is not specific

Pre-eclampsia is important as it predisposes to the following problems
fetal: prematurity, intrauterine growth retardation
eclampsia
haemorrhage: placental abruption, intra-abdominal, intra-cerebral
cardiac failure
multi-organ failure

Risk factors
> 40 years old
nulliparity (or new partner)
multiple pregnancy
body mass index > 30 kg/m^2
diabetes mellitus
pregnancy interval of more than 10 years
family history of pre-eclampsia
previous history of pre-eclampsia
pre-existing vascular disease such as hypertension or renal disease

Features of severe pre-eclampsia
hypertension: typically > 170/110 mmHg and proteinuria as above
proteinuria: dipstick ++/+++
headache
visual disturbance
papilloedema
RUQ/epigastric pain
hyperreflexia
platelet count 160/110 mmHg although many clinicians have a lower threshold
oral labetalol is now first-line following the 2010 NICE guidelines. Nifedipine and hydralazine may also be used
delivery of the baby is the most important and definitive management step. The timing depends on the individual clinical scenario

How well did you know this?
1
Not at all
2
3
4
5
Perfectly
22
Q

Which one of the following is an absolute contraindication to combined oral contraceptive pill use?

Controlled hypertension
History of cholestasis
36-year-old woman smoking 20 cigarettes/day
BMI of 38 kg/m^2
Migraine without aura
A

Combined oral contraceptive pill: contraindications

The decision of whether to start a women on the combined oral contraceptive pill is now guided by the UK Medical Eligibility Criteria (UKMEC). This scale categorises the potential cautions and contraindications according to a four point scale, as detailed below:
UKMEC 1: a condition for which there is no restriction for the use of the contraceptive method
UKMEC 2: advantages generally outweigh the disadvantages
UKMEC 3: disadvantages generally outweigh the advantages
UKMEC 4: represents an unacceptable health risk

Examples of UKMEC 3 conditions include
more than 35 years old and smoking less than 15 cigarettes/day
BMI > 35 kg/m^2*
migraine without aura and more than 35 years old
family history of thromboembolic disease in first degree relatives 20 years ago is classified as UKMEC 3 or 4 depending on severity

*The UKMEC 4 rating for a BMI > 40 kg/m^2 was removed in 2009.

How well did you know this?
1
Not at all
2
3
4
5
Perfectly
23
Q

You are reviewing test results. The midstream specimen of urine (MSU) from a 24-year-old woman who is 11 weeks pregnant shows a urinary tract infection. On discussing the result with the patient she does describe some dysuria and ‘smelly urine’. What is the most appropriate management?

Ciprofloxacin for 7 days
Amoxicillin for 7 days
Repeat MSU
Trimethoprim for 3 days
No treatment
A

As this woman is symptomatic she should be treated with an antibiotic that is safe to use in pregnancy.

Urinary tract infection in adults: management

Lower urinary tract infections in non-pregnant women
local antibiotic guidelines should be followed if available
2012 SIGN guidelines recommend trimethoprim or nitrofurantoin for 3 days

Pregnant women with symptomatic bacteriuria should be treated with an antibiotic for 7 days. A urine culture should be sent. For asymptomatic pregnant women:
a urine culture should be performed routinely at the first antenatal visit
if positive, a second urine culture should be sent to confirm the presence of bacteriuria
SIGN recommend to treat asymptomatic bacteriuria detected during pregnancy with an antibiotic
a 7 day course of antibiotics should be given
a further urine culture should be sent following completion of treatment as a test of cure

For patients with sign of acute pyelonephritis hospital admission should be considered
local antibiotic guidelines should be followed if available
the BNF currently recommends a broad-spectrum cephalosporin or a quinolone for 10-14 days

How well did you know this?
1
Not at all
2
3
4
5
Perfectly
24
Q

A 12-year-old girl comes with her mother to surgery. She is requesting the combined oral contraceptive (COC) pill as she has recently started a relationship with an 18-year-old man who she met at the local games arcade, although she has not yet had sex with him. You discuss the age difference but her mother states that she has met the boyfriend and says he treats her daughter well. Both the girl and her mother insist that they do not want anyone else involved. What is the most appropriate course of action?

Assess using Fraser guidelines + prescribe a Long Acting Reversible Contraceptive method such as Implanon
Assess using Fraser guidelines + prescribe the COC
Advise her to abstain and refer to a 'Safer Sex for Young Adults' program
Obtain consent from mother + prescribe the COC
Immediately phone local child protection lead and refer to social services
A

Children under the age of 13 years are not able to consent to sexual intercourse and hence any sexual activity would be regarded as rape under the law. This is one situation under the GMC guidelines where you are compelled to break confidentiality

Consent: children

The General Medical Council have produced guidelines on obtaining consent in children:
at 16 years or older a young person can be treated as an adult and can be presumed to have capacity to decide
under the age of 16 years children may have capacity to decide, depending on their ability to understand what is involved
where a competent child refuses treatment, a person with parental responsibility or the court may authorise investigation or treatment which is in the child’s best interests*

With regards to the provision of contraceptives to patients under 16 years of age the Fraser Guidelines state that all the following requirements should be fulfilled:
the young person understands the professional’s advice
the young person cannot be persuaded to inform their parents
the young person is likely to begin, or to continue having, sexual intercourse with or without contraceptive treatment
unless the young person receives contraceptive treatment, their physical or mental health, or both, are likely to suffer
the young person’s best interests require them to receive contraceptive advice or treatment with or without parental consent

Gillick or Fraser?

Some doctors use the term Fraser competency when referring to contraception and Gillick competency when referring to general issues of consent in children. The (widespread) rumours that Victoria Gillick removed her permission to use her name or applied copyright have recently been debunked.

More information can be found in the following article:

Wheeler R. Gillick or Fraser? A plea for consistency over competence in children BMJ 2006;332:807

*in Scotland those with parental responsibility cannot authorise procedures a competent child has refused

How well did you know this?
1
Not at all
2
3
4
5
Perfectly
25
Q

A woman presents to have a Nexplanon (etonogestrel) inserted. Where is the most appropriate place to insert the implant?

Subcutaneous, buttock
Subdermal, non-dominant arm
Subcutaneous, non-dominant arm
Intradermal, non-dominant arm
Subdermal, buttock
A

Implantable contraceptives

Implanon was a non-biodegradable subdermal contraceptive implant which has been replaced by Nexplanon. From a pharmacological perspective Nexplanon is the same as Implanon. The two main differences are:
the applicator has been redesigned to try and prevent ‘deep’ insertions (i.e. subcutaneous/intramuscular)
it is radiopaque and therefore easier to locate if impalpable

Both versions slowly releases the progestogen hormone etonogestrel. They are typically inserted in the proximal non-dominant arm, just overlying the tricep. The main mechanism of action is preventing ovulation. They also work by thickening the cervical mucus.

Key points
highly effective: failure rate 0.07/100 women/year
long-acting: lasts 3 years
doesn’t contain oestrogen so can be used if past history of thromboembolism, migraine etc
can be inserted immediately following a termination of pregnancy

Disadvantages include
the need for a trained professional to insert and remove device
additional contraceptive methods are needed for the first 7 days if not inserted on day 1 to 5 of a woman’s menstrual cycle

Adverse effects
irregular/heavy bleeding is the main problem
‘progestogen effects’: headache, nausea, breast pain

Interactions
enzyme-inducing drugs such as certain antiepileptic and rimampicin may reduce the efficacy of Nexplanon
the FSRH advises that women should be advised to switch to a method unaffected by enzyme-inducing drugs or to use additional contraception until 28 days after stopping the treatment

Contraindications
UKMEC 3*: ischaemic heart disease/stroke (for continuation, if initiation then UKMEC 2), unexplained, suspicious vaginal bleeding, past breast cancer, severe liver cirrhosis, liver cancer, positive antiphospholipid antibodies**
UKMEC 4**: current breast cancer

proven risks generally outweigh the advantages
**there is some contradiction in the guidance issued by the FSRH but their most recent document (revised 2010) lists positive antiphospholipid antibodies as UKMEC 3
**
a condition which represents an unacceptable risk if the contraceptive method is used

How well did you know this?
1
Not at all
2
3
4
5
Perfectly
26
Q

What is the failure rate of female sterilisation?

1 in 100
1 in 200
1 in 300
1 in 400
1 in 500
A

Sterilisation

Male sterilisation - vasectomy
failure rate: 1 per 2,000*
simple operation, can be done under LA (some GA), go home after a couple of hours
doesn’t work immediately
semen analysis needs to be performed twice following a vasectomy before a man can have unprotected sex (usually at 16 and 20 weeks)
complications: bruising, haematoma, infection, sperm granuloma, chronic testicular pain (affects between 5-30% men)
the success rate of vasectomy reversal is up to 55%, if done within 10 years, and approximately 25% after more than 10 years

Female sterilisation
failure rate: 1 per 200*
usually done by laparoscopy under general anaesthetic
generally done as a day case
many different techniques involving clips (e.g. Filshie clips) , blockage, rings (Falope rings) and salpingectomy
complications: increased risk of ectopic if sterilisation fails, general complications of GA/laparoscopy
the current success rate of female sterilisation reversal is between 50-60%

*source = Royal College of Obstetricians and Gynaecologists

How well did you know this?
1
Not at all
2
3
4
5
Perfectly
27
Q

How often is the Depo Provera (medroxyprogesterone acetate) injectable contraceptive given?

Every 2 weeks
Every 4 weeks
Every 8 weeks
Every 12 weeks
Every 24 weeks
A

Injectable contraceptives

Depo Provera is the main injectable contraceptive used in the UK*. It contains medroxyprogesterone acetate 150mg. It is given via in intramuscular injection every 12 weeks. It can however be given up to 14 weeks after the last dose without the need for extra precautions**

The main method of action is by inhibiting ovulation. Secondary effects include cervical mucus thickening and endometrial thinning.

Disadvantages include the fact that the injection cannot be reversed once given. There is also a potential delayed return to fertility (maybe up to 12 months)

Adverse effects
irregular bleeding
weight gain
may potentially increased risk of osteoporosis: should only be used in adolescents if no other method of contraception is suitable
not quickly reversible and fertility may return after a varying time

*Noristerat, the other injectable contraceptive licensed in the UK, is rarely used in clinical practice. It is given every 8 weeks

**the BNF gives different advice, stating a pregnancy test should be done if the interval is greater than 12 weeks and 5 days - this is however not commonly adhered to in the family planning community

How well did you know this?
1
Not at all
2
3
4
5
Perfectly
28
Q

Which one of the following statements regarding cervical ectropion is incorrect?

Describes an increased area of columnar epithelium
May result in post-coital bleeding
Is less common in women who use the combined oral contraceptive pill
May result in excessive vaginal discharge
Is more common during pregnancy
A

Cervical ectropion

On the ectocervix there is a transformation zone where the stratified squamous epithelium meets the columnar epithelium of the cervical canal. Elevated oestrogen levels (ovulatory phase, pregnancy, combined oral contraceptive pill use) result in larger area of columnar epithelium being present on the ectocervix

The term cervical erosion is used less commonly now

This may result in the following features
vaginal discharge
post-coital bleeding

Ablative treatment (for example ‘cold coagulation’) is only used for troublesome symptoms

How well did you know this?
1
Not at all
2
3
4
5
Perfectly
29
Q

Which one of the following is not a recognised adverse effect of the combined oral contraceptive pill?

Increased risk of ovarian cancer
Increased risk of deep vein thrombosis
Increased risk of breast cancer
Increased risk of ischaemic heart disease
Increased risk of cervical cancer
A

Combined oral contraceptive pill
increased risk of breast and cervical cancer
protective against ovarian and endometrial cancer

The combined oral contraceptive pill has actually been shown to reduce the risk of ovarian cancer

Combined oral contraceptive pill: advantages/disadvantages

Advantages of combined oral contraceptive pill
highly effective (failure rate

How well did you know this?
1
Not at all
2
3
4
5
Perfectly
30
Q

Which one of the following statements regarding the management of rhesus negative pregnancy is true?

A. It mainly causes problems for the mother’s first-born child
B. Anti-D is still required following delivery even if the mother received routine antenatal anti-D prophylaxis
C. Anti-D should be given within 72 hours of a spontaneous miscarriage at 9 weeks
D. Anti-D should be given as soon as the mother is found to be rhesus negative (usually around 10 weeks)
E. Coombs test is used to determine the rhesus status of the mother

A

B. Anti-D is still required following delivery even if the mother received routine antenatal ant-D prophylaxis

Subsequent pregnancies are most at risk following the sensitising event of the first childbirth.

Rhesus negative pregnancy

A basic understanding of the pathophysiology is essential to understand the management of Rhesus negative pregnancies
along with the ABO system the Rhesus system is the most important antigen found on red blood cells. The D antigen is the most important antigen of the rhesus system
around 15% of mothers are rhesus negative (Rh -ve)
if a Rh -ve mother delivers a Rh +ve child a leak of fetal red blood cells may occur
this causes anti-D IgG antibodies to form in mother
in later pregnancies these can cross placenta and cause haemolysis in fetus
this can also occur in the first pregnancy due to leaks

Prevention
test for D antibodies in all Rh -ve mothers at booking
NICE (2008) advise giving anti-D to non-sensitised Rh -ve mothers at 28 and 34 weeks
the evidence base suggests that there is little difference in the efficacy of single-dose (at 28 weeks) and double-dose regimes (at 28 & 34 weeks). For this reason the RCOG in 2011 advised that either regime could be used ‘depending on local factors’
anti-D is prophylaxis - once sensitization has occurred it is irreversible
if event is in 2nd/3rd trimester give large dose of anti-D and perform Kleihauer test - determines proportion of fetal RBCs present

Anti-D immunoglobulin should be given as soon as possible (but always within 72 hours) in the following situations:
delivery of a Rh +ve infant, whether live or stillborn
any termination of pregnancy
miscarriage if gestation is > 12 weeks
ectopic pregnancy
external cephalic version
antepartum haemorrhage
amniocentesis, chorionic villus sampling, fetal blood sampling

Tests
all babies born to Rh -ve mother should have cord blood taken at delivery for FBC, blood group & direct Coombs test
Coombs test: direct antiglobulin, will demonstrate antibodies on RBCs of baby
Kleihauer test: add acid to maternal blood, fetal cells are resistant

Affected fetus
oedematous (hydrops fetalis, as liver devoted to RBC production albumin falls)
jaundice, anaemia, hepatosplenomegaly
heart failure
kernicterus
treatment: transfusions, UV phototherapy

How well did you know this?
1
Not at all
2
3
4
5
Perfectly
31
Q

A 34-year-old woman presents requesting emergency contraception. She had unprotected sexual intercourse with her ex-partner four days ago and is very keen to avoid a pregnancy. Today she is 14 days into a 28 day cycle. Her past medical history includes an ectopic pregnancy around four years which resulted in a salpingectomy. Of the availalbe options, what is the most appropriate action?

Do a pregnancy test
Prescribe a stat dose of levonorgestrel 1.5mg but explain it may not be effective
Insert an intrauterine device
Explain she is outside the emergency contraception window and advise her take a pregnancy test if her period is late
Insert a intrauterine system (Mirena)
A

IUD - Previous ectopic pregnancy is not a contraindication to intrauterine device insertion. Intrauterine systems (Mirena) cannot be used for emergency contraception.

Emergency contraception

There are two methods currently available in the UK:

Emergency hormonal contraception

There are now two methods of emergency hormonal contraception (‘emergency pill’, ‘morning-after pill’); levonorgestrel and ulipristal, a progesterone receptor modulator.

Levonorgestrel
should be taken as soon as possible - efficacy decreases with time
must be taken within 72 hrs of unprotected sexual intercourse (UPSI)*
single dose of levonorgestrel 1.5mg (a progesterone)
mode of action not fully understood - acts both to stop ovulation and inhibit implantation
84% effective is used within 72 hours of UPSI
levonorgestrel is safe and well tolerated. Disturbance of the current menstrual cycle is seen in a significant minority of women. Vomiting occurs in around 1%
if vomiting occurs within 2 hours then the dose should be repeated
can be used more than once in a menstrual cycle if clinically indicated

Ulipristal
a progesterone receptor modulator currently marketed as EllaOne. The primary mode of action is thought to be inhibition of ovulation
30mg oral dose taken as soon as possible, no later than 120 hours after intercourse
concomitant use with levonorgestrel is not recommended
may reduce the effectiveness of combined oral contraceptive pills and progesterone only pills
caution should be exercised in patients with severe asthma
repeated dosing within the same menstrual cycle is not recommended
breastfeeding should be delayed for one week after taking ulipristal. There are no such restrictions on the use of levonorgestrel

Intrauterine device (IUD)
must be inserted within 5 days of UPSI, or
if a women presents after more than 5 days then an IUD may be fitted up to 5 days after the likely ovulation date
may inhibit fertilisation or implantation
prophylactic antibiotics may be given if the patient is considered to be at high-risk of sexually transmitted infection
is 99% effective regardless of where it is used in the cycle
may be left in-situ to provide long-term contraception. If the client wishes for the IUD to be removed it should be at least kept in until the next period

*may be offered after this period as long as the client is aware of reduced effectiveness and unlicensed indication

How well did you know this?
1
Not at all
2
3
4
5
Perfectly
32
Q

Which one of the following correctly describes the frequency of the cervical cancer screening program in England?

25-64 years - 5-yearly screening
20-49 years - 3-yearly screening; 50-69 years - 5-yearly screening
25-49 years - 5-yearly screening; 50-64 years - 3-yearly screening
25-64 years - 3-yearly screening
25-49 years - 3-yearly screening; 50-64 years - 5-yearly screening
A

Cervical cancer screening
25-49 years: 3-yearly
50-64 years: 5-yearly

The question is specific to England as there are some variations in practice across the UK

Cervical cancer screening

The UK has a well established cervical cancer screening program which is estimated to prevent 1,000-4,000 deaths per year. It should be noted that cervical adenocarcinomas, which account for around 15% of cases, are frequently undetected by screening

Who is screened and how often?

A smear test is offered to all women between the ages of 25-64 years
25-49 years: 3-yearly screening
50-64 years: 5-yearly screening

How is performed?

There is currently a move away from traditional Papanicolaou (Pap) smears to liquid-based cytology (LBC). Rather than smearing the sample onto a slide the sample is either rinsed into the preservative fluid or the brush head is simply removed into the sample bottle containing the preservative fluid.

Advantages of LBC includes
reduced rate of inadequate smears
increased sensitivity and specificity

It is said that the best time to take a cervical smear is around mid-cycle. Whilst there is limited evidence to support this it is still the current advice given out by the NHS.

In Scotland women from the ages of 20-60 years are screened every 3 years.

How well did you know this?
1
Not at all
2
3
4
5
Perfectly
33
Q

A 26-year-old woman presents due to heavy, painful periods. She typically has heavy bleeding for around 6 days with severe crampy pains for the first 3 days. She does not want to be on contraception as she is getting married shortly and plans to start a family. A full blood count is performed and is normal. What is the most appropriate first-line treatment?

Tranexamic acid
Paracetamol
Mefenamic acid
Norethisterone
Intrauterine system (Mirena)
A

Mefenamic acid - Tranexamic acid would not treat the dysmenorrhoea.

Menorrhagia: management

Menorrhagia was previously defined as total blood loss > 80 ml per menses, but it is obviously difficult to quantify. The management has therefore shifted towards what the woman considers to be excessive. Prior to the 1990’s many women underwent a hysterectomy to treat heavy periods but since that time the approach has altered radically. The management of menorrhagia now depends on whether a women needs contraception.

Investigations
a full blood count should be performed in all women
further investigations are based upon the history and examination findings

Does not require contraception
either mefenamic acid 500 mg tds (particularly if there is dysmenorrhoea as well) or tranexamic acid 1 g tds. Both are started on the first day of the period
if no improvement then try other drug whilst awaiting referral

Requires contraception, options include
intrauterine system (Mirena) should be considered first-line
combined oral contraceptive pill
long-acting progestogens

Norethisterone 5 mg tds can be used as a short-term option to rapidly stop heavy menstrual bleeding.

How well did you know this?
1
Not at all
2
3
4
5
Perfectly
34
Q

A woman who is 36 weeks pregnant is reviewed. This is her first pregnancy. Her baby is known to currently lie in a breech presentation. What is the most appropriate management?

Reassure mother baby will most likely turn to a cephalic presentation prior to delivery
Refer for external cephalic version
Admit for induction of labour and trial of vaginal delivery
Refer for radiological pelvimetry
Admit for caesarean section
A

Refer for external cephalic version

Breech presentation

In a breech presentation the caudal end of the fetus occupies the lower segment. Whilst around 25% of pregnancies at 28 weeks are breech it only occurs in 3% of babies near term. A frank breech is the most common presentation with the hips flexed and knees fully extended. A footling breech, where one or both feet come first with the bottom at a higher position, is rare but carries a higher perinatal morbidity

Risk factors for breech presentation
uterine malformations, fibroids
placenta praevia
polyhydramnios or oligohydramnios
fetal abnormality (e.g. CNS malformation, chromosomal disorders)
prematurity (due to increased incidence earlier in gestation)

Cord prolapse is more common in breech presentations

Management
if

How well did you know this?
1
Not at all
2
3
4
5
Perfectly
35
Q

Which one of the following statements is not correct regarding hypertension in pregnancy?

A. An increase above booking readings of > 30 mmHg systolic or > 15 mmHg diastolic suggests hypertension
B. Pre-eclampsia occurs in around 5% of pregnancies
C. Urine dipstick showing protein + is consistent with gestational hypertension
D. A rise in blood pressure before 20 weeks suggests pre-existing hypertension
E. With gestational hypertension the blood pressure rises in the second half of pregnancy

A

C. Proteinuria suggests pre-eclampsia

Hypertension in pregnancy

NICE published guidance in 2010 on the management of hypertension in pregnancy. They also made recommendations on reducing the risk of hypertensive disorders developing in the first place. Women who are at high risk of developing pre-eclampsia should take aspirin 75mg od from 12 weeks until the birth of the baby. High risk groups include:
hypertensive disease during previous pregnancies
chronic kidney disease
autoimmune disorders such as SLE or antiphospholipid syndrome
type 1 or 2 diabetes mellitus

The classification of hypertension in pregnancy is complicated and varies. Remember, in normal pregnancy:
blood pressure usually falls in the first trimester (particularly the diastolic), and continues to fall until 20-24 weeks
after this time the blood pressure usually increases to pre-pregnancy levels by term

Hypertension in pregnancy in usually defined as:
systolic > 140 mmHg or diastolic > 90 mmHg
or an increase above booking readings of > 30 mmHg systolic or > 15 mmHg diastolic

How well did you know this?
1
Not at all
2
3
4
5
Perfectly
36
Q

Which one of the following statements regarding polycystic ovarian syndrome (PCOS) is incorrect?

A slightly elevated prolactin is consistent with a diagnosis of PCOS
Luteinizing hormone levels are usually raised
Hyperinsulinaemia is seen
Acanthosis nigricans may be seen
Affects around 1% of women of reproductive age
A

Polycystic ovarian syndrome is extremely common, affecting between 5-20% of women of reproductive age

Polycystic ovarian syndrome: features and investigation

Polycystic ovary syndrome (PCOS) is a complex condition of ovarian dysfunction thought to affect between 5-20% of women of reproductive age. The aetiology of PCOS is not fully understood. Both hyperinsulinaemia and high levels of luteinizing hormone are seen in PCOS and there appears to be some overlap with the metabolic syndrome.

Features
subfertility and infertility
menstrual disturbances: oligomenorrhea and amenorrhoea
hirsutism, acne (due to hyperandrogenism)
obesity
acanthosis nigricans (due to insulin resistance)

Investigations
pelvic ultrasound: multiple cysts on the ovaries
FSH, LH, prolactin, TSH, and testosterone are useful investigations: raised LH:FSH ratio is a ‘classical’ feature but is no longer thought to be useful in diagnosis. Prolactin may be normal or mildly elevated. Testosterone may be normal or mildly elevated - however, if markedly raised consider other causes
check for impaired glucose tolerance

How well did you know this?
1
Not at all
2
3
4
5
Perfectly
37
Q

Theme: Menopause: management

A. Clonidine
B. Hormone replacement therapy
C. Tibolone
D. Topical oestrogens
E. Topical steroids
F. Selective serotonin reuptake inhibitor
G. Black cohosh
H. Raloxifene

For each one of the following select the most appropriate answer from the options above:

Useful for vaginal atrophy

A

Topical oestrogens

Menopause: management

Many women have little or no symptoms around the menopause do not require any specific treatment other than advice and reassurance. Hormone replacement therapy (HRT) should be used primarily for the treatment of menopausal symptoms. It should no longer be given in an attempt to modify cardiovascular risk (following the Women’s Health Initiative Study) but may be beneficial in the prevention and treatment of osteoporosis.

Management options for hot flushes or night sweats
lifestyle advice: exercise, avoiding caffeine/spicy foods, lighter clothing
hormone replacement therapy: most effective
tibolone: unsuitable for use within 12 months of last menstrual period as may cause irregular bleeding
clonidine: use is often limited by side-effects such as dry mouth, dizziness and nausea
selective serotonin reuptake inhibitors: only small trials have been completed to date

Vaginal atrophy
topical oestrogens

How well did you know this?
1
Not at all
2
3
4
5
Perfectly
38
Q

Theme: Menopause: management

A. Clonidine
B. Hormone replacement therapy
C. Tibolone
D. Topical oestrogens
E. Topical steroids
F. Selective serotonin reuptake inhibitor
G. Black cohosh
H. Raloxifene

For each one of the following select the most appropriate answer from the options above:

Unsuitable for use within 12 months of last menstrual period as may cause irregular bleeding

A

Tibolone

Menopause: management

Many women have little or no symptoms around the menopause do not require any specific treatment other than advice and reassurance. Hormone replacement therapy (HRT) should be used primarily for the treatment of menopausal symptoms. It should no longer be given in an attempt to modify cardiovascular risk (following the Women’s Health Initiative Study) but may be beneficial in the prevention and treatment of osteoporosis.

Management options for hot flushes or night sweats
lifestyle advice: exercise, avoiding caffeine/spicy foods, lighter clothing
hormone replacement therapy: most effective
tibolone: unsuitable for use within 12 months of last menstrual period as may cause irregular bleeding
clonidine: use is often limited by side-effects such as dry mouth, dizziness and nausea
selective serotonin reuptake inhibitors: only small trials have been completed to date

Vaginal atrophy
topical oestrogens

How well did you know this?
1
Not at all
2
3
4
5
Perfectly
39
Q

Theme: Menopause: management

A. Clonidine
B. Hormone replacement therapy
C. Tibolone
D. Topical oestrogens
E. Topical steroids
F. Selective serotonin reuptake inhibitor
G. Black cohosh
H. Raloxifene

For each one of the following select the most appropriate answer from the options above:

Often causes dry mouth and dizziness

A

Clonidine

Menopause: management

Many women have little or no symptoms around the menopause do not require any specific treatment other than advice and reassurance. Hormone replacement therapy (HRT) should be used primarily for the treatment of menopausal symptoms. It should no longer be given in an attempt to modify cardiovascular risk (following the Women’s Health Initiative Study) but may be beneficial in the prevention and treatment of osteoporosis.

Management options for hot flushes or night sweats
lifestyle advice: exercise, avoiding caffeine/spicy foods, lighter clothing
hormone replacement therapy: most effective
tibolone: unsuitable for use within 12 months of last menstrual period as may cause irregular bleeding
clonidine: use is often limited by side-effects such as dry mouth, dizziness and nausea
selective serotonin reuptake inhibitors: only small trials have been completed to date

Vaginal atrophy
topical oestrogens

How well did you know this?
1
Not at all
2
3
4
5
Perfectly
40
Q

A 31-year-old female presents to the genitourinary medicine clinic due to four fleshy, protuberant lesions on her vulva which are slightly pigmented. She has recently started a relationship with a new partner. What is the most appropriate initial management?

Oral aciclovir
Topical podophyllum
Topical salicylic acid
Topical aciclovir
Electrocautery
A

Genital wart treatment
multiple, non-keratinised warts: topical podophyllum
solitary, keratinised warts: cryotherapy

Cryotherapy is also acceptable as an initial treatment for genital warts (see below)

Genital warts

Genital warts (also known as condylomata accuminata) are a common cause of attendance at genitourinary clinics. They are caused by the many varieties of the human papilloma virus HPV, especially types 6 & 11. It is now well established that HPV (primarily types 16,18 & 33) predisposes to cervical cancer.

Features
small (2 - 5 mm) fleshy protuberances which are slightly pigmented
may bleed or itch

Management
topical podophyllum or cryotherapy are commonly used as first-line treatments depending on the location and type of lesion. Multiple, non-keratinised warts are generally best treated with topical agents whereas solitary, keratinised warts respond better to cryotherapy
imiquimod is a topical cream which is generally used second line
genital warts are often resistant to treatment and recurrence is common although the majority of anogenital infections with HPV clear without intervention within 1-2 years

How well did you know this?
1
Not at all
2
3
4
5
Perfectly
41
Q

A woman who is 14 weeks pregnant presents as she came into contact with a child who has chickenpox around 4 days ago. She is unsure if she had the condition herself as a child. Blood tests show the following:

Varicella IgM Negative
Varicella IgG Negative

What is the most appropriate management?

Varicella zoster immunoglobulin
No action required
IV aciclovir
Varicella zoster vaccination
Varicella zoster vaccination + varicella zoster immunoglobulin
A

Chickenpox exposure in pregnancy - if not immune give VZIG

The negative IgG indicates no previous exposure to chickenpox

Chickenpox exposure in pregnancy

Chickenpox is caused by primary infection with varicella zoster virus. Shingles is reactivation of dormant virus in dorsal root ganglion. In pregnancy there is a risk to both the mother and also the fetus, a syndrome now termed fetal varicella syndrome

Fetal varicella syndrome (FVS)
risk of FVS following maternal varicella exposure is around 1% if occurs before 20 weeks gestation
studies have shown a very small number of cases occurring between 20-28 weeks gestation and none following 28 weeks
features of FVS include skin scarring, eye defects (microphthalmia), limb hypoplasia, microcephaly and learning disabilities

Management of chickenpox exposure
if there is any doubt about the mother previously having chickenpox maternal blood should be checked for varicella antibodies
if the pregnant women is not immune to varicella she should be given varicella zoster immunoglobulin (VZIG) as soon as possible. RCOG and Greenbook guidelines suggest VZIG is effective up to 10 days post exposure
consensus guidelines suggest oral aciclovir should be given if pregnant women with chickenpox present within 24 hours of onset of the rash

How well did you know this?
1
Not at all
2
3
4
5
Perfectly
42
Q

A 44-year-old female has a Mirena (intrauterine system) fitted for contraception on day 12 of her cycle. How long will it take before it can be relied upon as a method of contraception?

Immediately
2 days
5 days
7 days
Until first day of next period
A

Contraceptives - time until effective (if not first day period):
instant: IUD
2 days: POP
7 days: COC, injection, implant, IUS

Intrauterine contraceptive devices

Intrauterine contraceptive devices comprise both conventional copper intrauterine devices (IUDs) and levonorgestrel-releasing intrauterine systems (IUS, Mirena). The IUS is also used in the management of menorrhagia

Effectiveness
both the IUD and IUS are more than 99% effective

Mode of action
IUD: primary mode of action is prevention of fertilisation by causing decreased sperm motility and survival (possibly an effect of copper ions)
IUS: levonorgestrel prevents endometrial proliferation and causes cervical mucous thickening

Counselling
IUD is effective immediately following insertion
IUS can be relied upon after 7 days

Potential problems
IUDs make periods heavier, longer and more painful
the IUS is associated with initial frequent uterine bleeding and spotting. Later women typically have intermittent light menses with less dysmenorrhoea and some women become amenorrhoeic
uterine perforation: up to 2 per 1000 insertions
the proportion of pregnancies that are ectopic is increased but the absolute number of ectopic pregnancies is reduced, compared to a woman not using contraception
infection: there is a small increased risk of pelvic inflammatory disease in the first 20 days after insertion but after this period the risk returns to that of a standard population
expulsion: risk is around 1 in 20, and is most likely to occur in the first 3 months

How well did you know this?
1
Not at all
2
3
4
5
Perfectly
43
Q

Which one of the following statements regarding the management of diabetes mellitus during pregnancy is incorrect?

A previous macrosomic baby is a risk factor for gestational diabetes
Diabetes complicates around 1 in 40 pregnancies
A higher dose of folic acid (5 mg/day) should be used
Metformin is contraindicated
Tight glycaemic control reduces complication rates
A

There is increasing evidence that metformin is safe during pregnancy

Pregnancy: diabetes mellitus

Diabetes mellitus may be a pre-existing problem or develop during pregnancy, gestational diabetes. It complicates around 1 in 40 pregnancies

Risk factors for gestational diabetes
BMI of > 30 kg/m^2
previous macrosomic baby weighing 4.5 kg or above.
previous gestational diabetes
first-degree relative with diabetes
family origin with a high prevalence of diabetes (South Asian, black Caribbean and Middle Eastern)

Screening for gestational diabetes
if a women has had gestational diabetes previously an oral glucose tolerance test (OGTT) should be performed at 16-18 weeks and at 28 weeks if the first test is normal
women with any of the other risk factors should be offered an OGTT at 24-28 weeks
currently the same WHO diagnostic criteria are used as for non-pregnant patients. There is however increasing evidence that a lower threshold should be used as treating borderline patients improves both maternal and neonatal outcomes

NICE issued guidelines on the management of diabetes mellitus in pregnancy which were updated in 2008

Management of pre-existing diabetes
weight loss for women with BMI of > 27 kg/m^2
stop oral hypoglycaemic agents, apart from metformin, and commence insulin
folic acid 5 mg/day from pre-conception to 12 weeks gestation
detailed anomaly scan at 18-20 weeks including four-chamber view of the heart and outflow tracts
tight glycaemic control reduces complication rates
treat retinopathy as can worsen during pregnancy

Management of gestational diabetes
responds to changes in diet and exercise in around 80% of women
oral hypoglycaemic agents (metformin or glibenclamide) or insulin injections are needed if blood glucose control is poor or this is any evidence of complications (e.g. macrosomia)
there is increasing evidence that oral hypoglycaemic agents are both safe and give similar outcomes to insulin
hypoglycaemic medication should be stopped following delivery
a fasting glucose should be checked at the 6 week postnatal check

How well did you know this?
1
Not at all
2
3
4
5
Perfectly
44
Q

A 27-year-old female asks for advice regarding the Mirena (intrauterine system). What is the most likely affect on her periods?

Continual, light bleeding is seen in 70%
Initially irregular bleeding later followed by periods that are generally heavier and longer
Heavy period approximately every 3 months
Amenorrhoea in > 90% after 2 months
Initially irregular bleeding later followed by light menses or amenorrhoea
A

Initially irregular bleeding later followed by light menses or amenorrhoea

Intrauterine contraceptive devices

Intrauterine contraceptive devices comprise both conventional copper intrauterine devices (IUDs) and levonorgestrel-releasing intrauterine systems (IUS, Mirena). The IUS is also used in the management of menorrhagia

Effectiveness
both the IUD and IUS are more than 99% effective

Mode of action
IUD: primary mode of action is prevention of fertilisation by causing decreased sperm motility and survival (possibly an effect of copper ions)
IUS: levonorgestrel prevents endometrial proliferation and causes cervical mucous thickening

Counselling
IUD is effective immediately following insertion
IUS can be relied upon after 7 days

Potential problems
IUDs make periods heavier, longer and more painful
the IUS is associated with initial frequent uterine bleeding and spotting. Later women typically have intermittent light menses with less dysmenorrhoea and some women become amenorrhoeic
uterine perforation: up to 2 per 1000 insertions
the proportion of pregnancies that are ectopic is increased but the absolute number of ectopic pregnancies is reduced, compared to a woman not using contraception
infection: there is a small increased risk of pelvic inflammatory disease in the first 20 days after insertion but after this period the risk returns to that of a standard population
expulsion: risk is around 1 in 20, and is most likely to occur in the first 3 months

How well did you know this?
1
Not at all
2
3
4
5
Perfectly
45
Q

A 22-year-old female has a Nexplanon inserted. For how long will this provide effective contraception?

12 weeks
12 months
3 years
5 years
7 years
A

3 Years

Implantable contraceptives

Implanon was a non-biodegradable subdermal contraceptive implant which has been replaced by Nexplanon. From a pharmacological perspective Nexplanon is the same as Implanon. The two main differences are:
the applicator has been redesigned to try and prevent ‘deep’ insertions (i.e. subcutaneous/intramuscular)
it is radiopaque and therefore easier to locate if impalpable

Both versions slowly releases the progestogen hormone etonogestrel. They are typically inserted in the proximal non-dominant arm, just overlying the tricep. The main mechanism of action is preventing ovulation. They also work by thickening the cervical mucus.

Key points
highly effective: failure rate 0.07/100 women/year
long-acting: lasts 3 years
doesn’t contain oestrogen so can be used if past history of thromboembolism, migraine etc
can be inserted immediately following a termination of pregnancy

Disadvantages include
the need for a trained professional to insert and remove device
additional contraceptive methods are needed for the first 7 days if not inserted on day 1 to 5 of a woman’s menstrual cycle

Adverse effects
irregular/heavy bleeding is the main problem
‘progestogen effects’: headache, nausea, breast pain

Interactions
enzyme-inducing drugs such as certain antiepileptic and rimampicin may reduce the efficacy of Nexplanon
the FSRH advises that women should be advised to switch to a method unaffected by enzyme-inducing drugs or to use additional contraception until 28 days after stopping the treatment

Contraindications
UKMEC 3*: ischaemic heart disease/stroke (for continuation, if initiation then UKMEC 2), unexplained, suspicious vaginal bleeding, past breast cancer, severe liver cirrhosis, liver cancer, positive antiphospholipid antibodies**
UKMEC 4**: current breast cancer

proven risks generally outweigh the advantages
**there is some contradiction in the guidance issued by the FSRH but their most recent document (revised 2010) lists positive antiphospholipid antibodies as UKMEC 3
**
a condition which represents an unacceptable risk if the contraceptive method is used

How well did you know this?
1
Not at all
2
3
4
5
Perfectly
46
Q

A baby is born to a mother who is known to have chronic hepatitis B. The mothers latest results are as follows:

HBsAg Positive
HBeAg Positive

What is the most appropriate strategy for reducing the vertical transmission rate?

Give the newborn hepatitis B vaccine + hepatitis B immunoglobulin
Give the newborn hepatitis B vaccine
Give the newborn hepatitis B immunoglobulin
Give the mother intravenous zidovudine during labour
Give the mother hepatitis B immunoglobulin shortly before birth + the newborn hepatitis B vaccine
A

Hepatitis B and pregnancy

Basics
all pregnant women are offered screening for hepatitis B
babies born to mothers who are chronically infected with hepatitis B or to mothers who’ve had acute hepatitis B during pregnancy should receive a complete course of vaccination + hepatitis B immunoglobulin
studies are currently evaluating the role of oral antiviral treatment (e.g. Lamivudine) in the latter part of pregnancy
there is little evidence to suggest caesarean section reduces vertical transmission rates
hepatitis B cannot be transmitted via breastfeeding (in contrast to HIV)

How well did you know this?
1
Not at all
2
3
4
5
Perfectly
47
Q

A 41-year-old female is diagnosed as having Trichomonas vaginalis. What is the most appropriate management?

Clotrimazole pessary
Oral doxycycline
Advice regarding hygiene and cotton underwear
Oral itraconazole
Oral metronidazole
A

Trichomonas vaginalis

Trichomonas vaginalis is a highly motile, flagellated protozoan parasite

Features
vaginal discharge: offensive, yellow/green, frothy
vulvovaginitis
strawberry cervix
pH > 4.5
in men is usually asymptomatic but may cause urethritis

Investigation
microscopy of a wet mount shows motile trophozoites

Management
oral metronidazole for 5-7 days, although the BNF also supports the use of a one-off dose of 2g metronidazole

How well did you know this?
1
Not at all
2
3
4
5
Perfectly
48
Q

A 25-year-old female presents to her GP as she has missed two consecutive Microgynon 30 pills. She has taken the Microgynon for the past 5 years and is currently 11 days into a packet of pills. Last night she had sexual intercourse with a new partner but unfortunately the condom split. What is the correct management?

Perform a pregnancy test in two days time
Advise the women she may be at risk from pregnancy and refer give a patient information leaflet explaining her options
Advise condom use for the next 7 days
Give a Depo-Provera (depot medroxyprogesterone acetate) injection to prevent implantation
Omit pill break at end of pack
A

Updated guidance from the FSRH states the following after a woman has missed two pills:

‘If you have missed two or more pills (i.e. more than 48 hours late), anywhere in the pack continue taking the rest of the pack as usual and use an extra method of contraception for the next 7 days’

For further information please consult the link to the FSRH guidelines. As this was a new partner consideration should be given to STI screening after an appropriate interval.

According to the FSRH emergency contraception does not need to be given in this scenario.

Depo-Provera is a long-acting contraceptive. It should not be used for emergency contraception and not given unless there is no chance of pregnancy.

Combined oral contraceptive pill: missed pill

The advice from the Faculty of Sexual and Reproductive Healthcare (FSRH) has changed over recent years. The following recommendations are now made for women taken a combined oral contraceptive (COC) pill containing 30-35 micrograms of ethinylestradiol

If 1 pill is missed (at any time in the cycle)
take the last pill even if it means taking two pills in one day and then continue taking pills daily, one each day
no additional contraceptive protection needed

If 2 or more pills missed
take the last pill even if it means taking two pills in one day, leave any earlier missed pills and then continue taking pills daily, one each day
the women should use condoms or abstain from sex until she has taken pills for 7 days in a row. FSRH: ‘This advice may be overcautious in the second and third weeks, but the advice is a backup in the event that further pills are missed’
if pills are missed in week 1 (Days 1-7): emergency contraception should be considered if she had unprotected sex in the pill-free interval or in week 1
if pills are missed in week 2 (Days 8-14): after seven consecutive days of taking the COC there is no need for emergency contraception*
if pills are missed in week 3 (Days 15-21): she should finish the pills in her current pack and start a new pack the next day; thus omitting the pill free interval

*theoretically women would be protected if they took the COC in a pattern of 7 days on, 7 days off

How well did you know this?
1
Not at all
2
3
4
5
Perfectly
49
Q

A couple present to the surgery as they are having trouble conceiving. They have been having regular unprotected sex for the past 12 months. The woman is 27-years-old and has a regular 28 day cycle. The man has no past history of note. What is the most appropriate initial management?

Check day 21 progesterone + semen analysis
Reassurance
Check day 14 progesterone + semen analysis
Referral to infertility services
Check day 14 progesterone + semen analysis + testosterone level in male
A

It should be noted that whilst NICE define infertility as ‘failure to conceive after regular unprotected sexual intercourse for 2 years in the absence of known reproductive pathology’ they advise that investigations are offered after 1 year

Infertility

Infertility affects around 1 in 7 couples. Around 84% of couples who have regular sex will conceive within 1 year, and 92% within 2 years

Causes
male factor 30%
unexplained 20%
ovulation failure 20%
tubal damage 15%
other causes 15%

Basic investigations
semen analysis
serum progesterone 7 days prior to expected next period

Key counselling points
folic acid
aim for BMI 20-25
advise regular sexual intercourse every 2 to 3 days
smoking/drinking advice

How well did you know this?
1
Not at all
2
3
4
5
Perfectly
50
Q

A pregnant woman in her 35th week of gestation goes in to spontaneous labour. Which one of the following is least associated with preterm birth?

Antepartum haemorrhage
Uterine abnormalities
Cervical incompetence
Endometriosis
Twin pregnancy
A

Endometriosis

Preterm birth

Preterm birth is defined as delivery of an infant before 37 weeks gestation. It occurs in around 5-10% of pregnancies (6% of singletons, 45% of twins)

Causes
unexplained (30-40%)
multiple pregnancies (20-30%)
congenital abnormalities
antepartum haemorrhage
pre-eclampsia
cervical incompetence
diabetes mellitus
polyhydramnios
uterine abnormalities
infections e.g. Pyelonephritis

How well did you know this?
1
Not at all
2
3
4
5
Perfectly
51
Q

Which one of the following statements regarding uterine fibroids is incorrect?

Are benign smooth muscle tumours of the uterus
May cause urinary symptoms
Undergo malignant change in 1 in 200 cases
Occur in around 20% of white women
Are more common in black women
A

The majority of pathologists believe that uterine leiomyosarcomas which may initially present as ‘fibroids’ are probably de novo lesions rather than a transformation of existing fibroids

Uterine fibroids

Fibroids are benign smooth muscle tumours of the uterus. They are through to occur in around 20% of white and around 50% of black women in the later reproductive years

Associations
more common in Afro-Caribbean women
rare before puberty, develop in response to oestrogen, don’t tend to progress following menopause

Features
may be asymptomatic
menorrhagia
bloating
urinary symptoms, e.g. frequency, may occur with larger fibroids
subfertility

Management
medical: symptomatic management e.g. with combined oral contraceptive pill. GnRH agonists may reduce the size of the fibroid but are typically useful for short-term treatment
surgery is sometimes needed
uterine artery embolization

Complications
red degeneration - haemorrhage into tumour - commonly occurs during pregnancy

How well did you know this?
1
Not at all
2
3
4
5
Perfectly
52
Q

A 25-year-old woman comes to surgery as she has not had a normal period for the past 12 months. She has done a pregnancy test recently which was negative. You arrange baseline investigations. Which one of the following is not part of your differential diagnosis?

Microprolactinoma
Polycystic ovarian syndrome
Turner's syndrome
Premature ovarian failure
Thyrotoxicosis
A

Turner’s syndrome causes primary rather than secondary amenorrhoea.

Amenorrhoea

Amenorrhoea may be divided into primary (failure to start menses by the age of 16 years) or secondary (cessation of established, regular menstruation for 6 months or longer).

Causes of primary amenorrhoea
Turner’s syndrome
testicular feminisation
congenital adrenal hyperplasia
congenital malformations of the genital tract

Causes of secondary amenorrhoea (after excluding pregnancy)
hypothalamic amenorrhoea (e.g. Stress, excessive exercise)
polycystic ovarian syndrome (PCOS)
hyperprolactinaemia
premature ovarian failure
thyrotoxicosis*

Initial investigations
exclude pregnancy with urinary or serum bHCG
gonadotrophins: low levels indicate a hypothalamic cause where as raised levels suggest an ovarian problem (e.g. Premature ovarian failure)
prolactin
androgen levels: raised levels may be seen in PCOS
oestradiol
thyroid function tests

*hypothyroidism may also cause amenorrhoea

How well did you know this?
1
Not at all
2
3
4
5
Perfectly
53
Q

What is the most common adverse effect experienced by women taking the progestogen only pill?

Irregular vaginal bleeding
Acne
Mood swings
Reduced libido
Weight gain
A

Irregular vaginal bleeding

Progestogen only pill: advantages/disadvantages

Advantages
highly effective (failure rate = 1 per 100 woman years)
doesn’t interfere with sex
contraceptive effects reversible upon stopping
can be used whilst breast-feeding
can be used in situations where the combined oral contraceptive pill is contraindicated e.g. in smokers > 35 years of age and women with a history of venous thromboembolic disease

Disadvantages
irregular periods: some users may not have periods whilst others may have irregular or light periods. This is the most common adverse effect
doesn’t protect against sexually transmitted infections
increased incidence of functional ovarian cysts
common side-effects include breast tenderness, weight gain, acne and headaches. These symptoms generally subside after the first few months

How well did you know this?
1
Not at all
2
3
4
5
Perfectly
54
Q

A 12-year-old girl comes to surgery with her mother who requests more information about human papilloma virus (HPV) vaccination. Which one of the following statements regarding HPV vaccination is incorrect?

Gardasil is given in 2 doses
HPV is the main aetiological factor in the development of cervical cancer
Gardasil protects against HPV 6, 11, 16 & 18
A vaccination program has been introduced for girls aged 12-13 years
Cervarix has the advantage over Gardasil of offering protection against genital warts
A

Gardasil, rather than Cervarix, has the advantage of offering protection against genital warts.

Human papilloma virus vaccination

It has been known for a longtime that the human papilloma virus (HPV) which infects the keratinocytes of the skin and mucous membranes is carcinogenic.

There are dozens of strains of HPV. The most important to remember are:
6 & 11: causes genital warts
16 & 18: linked to a variety of cancers, most notably cervical cancer

HPV infection is linked to:
over 99.7% of cervical cancers
around 85% of anal cancers
around 50% of vulval and vaginal cancers
around 20-30% of mouth and throat cancers

It should of course be remembered that there are other risk factors important in developing cervical cancer such as smoking, combined oral contraceptive pill use and high parity.

Testing for HPV has now been integrated into the cervical cancer screening programme. If a smear is reported as borderline or mild dyskaryosis the original sample is tested for HPV
if HPV negative the patient goes back to routine recall
if HPV positive the patient is referred for colposcopy

Immunisation

A vaccination for HPV was introduced in the UK back in 2008. As you may remember the Department of Health initially chose Cervarix. This vaccine protected against HPV 16 & 18 but not 6 & 11. There was widespread criticism of this decision given the significant disease burden caused by genital warts. Eventually in 2012 Gardasil replaced Cervarix as the vaccine used. Gardasil protects against HPV 6, 11, 16 & 18.

Girls aged 12-13 years are offered the vaccine in the UK
the vaccine is normally given in school
information given to parents and available on the NHS website make it clear that the daughter may receive the vaccine against parental wishes
given as 2 doses - girls have the second dose between 6-24 months after the first, depending on local policy

Injection site reactions are particularly common with HPV vaccines.

How well did you know this?
1
Not at all
2
3
4
5
Perfectly
55
Q

A 36-year-old female starts Cerazette (desogestrel) on day 7 of her cycle. How long will it take before it can be relied upon as a method of contraception?

Immediately
2 days
5 days
7 days
Until first day of next period
A

Contraceptives - time until effective (if not first day period):
instant: IUD
2 days: POP
7 days: COC, injection, implant, IUS

Progestogen only pill: counselling

Women who are considering taking the progestogen only pill (POP) should be counselled in a number of areas:

Potential adverse effects
irregular vaginal bleeding is the most common problem

Starting the POP
if commenced up to and including day 5 of the cycle it provides immediate protection, otherwise additional contraceptive methods (e.g. Condoms) should be used for the first 2 days
if switching from a combined oral contraceptive (COC) gives immediate protection if continued directly from the end of a pill packet (i.e. Day 21)

Taking the POP
should be taken at same time everyday, without a pill free break (unlike the COC)

Missed pills
if 3 hours*: take missed pill as soon as possible, continue with rest of pack, extra precautions (e.g. Condoms) should be used until pill taking has been re-established for 48 hours

Other potential problems
diarrhoea and vomiting: continue taking POP but assume pills have been missed - see above
antibiotics: have no effect on the POP**
liver enzyme inducers may reduce effectiveness

Other information
discussion on STIs

*for Cerazette (desogestrel) a 12 hour period is allowed

**unless the antibiotic alters the P450 enzyme system, for example rifampicin

How well did you know this?
1
Not at all
2
3
4
5
Perfectly
56
Q

A 29-year-old woman presents to the genitourinary medicine clinic for treatment of recurrent genital warts. Which one the following viruses are most likely to be responsible?

Human papilloma virus 16 & 18
Human papilloma virus 13 & 17
Human papilloma virus 6 & 11
Human papilloma virus 12 & 14
Human papilloma virus 15 & 21
A

Genital warts - 90% are caused by HPV 6 & 11

Types 6 and 11 are responsible for 90% of genital warts cases

Genital warts

Genital warts (also known as condylomata accuminata) are a common cause of attendance at genitourinary clinics. They are caused by the many varieties of the human papilloma virus HPV, especially types 6 & 11. It is now well established that HPV (primarily types 16,18 & 33) predisposes to cervical cancer.

Features
small (2 - 5 mm) fleshy protuberances which are slightly pigmented
may bleed or itch

Management
topical podophyllum or cryotherapy are commonly used as first-line treatments depending on the location and type of lesion. Multiple, non-keratinised warts are generally best treated with topical agents whereas solitary, keratinised warts respond better to cryotherapy
imiquimod is a topical cream which is generally used second line
genital warts are often resistant to treatment and recurrence is common although the majority of anogenital infections with HPV clear without intervention within 1-2 years

How well did you know this?
1
Not at all
2
3
4
5
Perfectly
57
Q

A 38-year-old female develops hypertension in the third trimester of her first pregnancy. A 24 hour urine collection shows 0.5g protein. Which one of the following complications is least associated with this condition?

Intracerebral haemorrhage
Pulmonary oedema
Fetal prematurity
Transverse myelitis
Fetal intrauterine growth retardation
A

Transverse myelitis is not associated with pre-eclampsia

Pre-eclampsia

Pre-eclampsia is a condition seen after 20 weeks gestation characterised by pregnancy-induced hypertension in association with proteinuria (> 0.3g / 24 hours). Oedema used to be third element of the classic triad but is now often not included in the definition as it is not specific

Pre-eclampsia is important as it predisposes to the following problems
fetal: prematurity, intrauterine growth retardation
eclampsia
haemorrhage: placental abruption, intra-abdominal, intra-cerebral
cardiac failure
multi-organ failure

Risk factors
> 40 years old
nulliparity (or new partner)
multiple pregnancy
body mass index > 30 kg/m^2
diabetes mellitus
pregnancy interval of more than 10 years
family history of pre-eclampsia
previous history of pre-eclampsia
pre-existing vascular disease such as hypertension or renal disease

Features of severe pre-eclampsia
hypertension: typically > 170/110 mmHg and proteinuria as above
proteinuria: dipstick ++/+++
headache
visual disturbance
papilloedema
RUQ/epigastric pain
hyperreflexia
platelet count 160/110 mmHg although many clinicians have a lower threshold
oral labetalol is now first-line following the 2010 NICE guidelines. Nifedipine and hydralazine may also be used
delivery of the baby is the most important and definitive management step. The timing depends on the individual clinical scenario

How well did you know this?
1
Not at all
2
3
4
5
Perfectly
58
Q

A 62-year-old woman presents with abdominal bloating. Examination reveals shifting dullness. Which one of the following is a risk factor for ovarian cancer?

Combined oral contraceptive pill use
Late menarche
Having many children
Early menopause
BRCA2 gene
A

BRCA2 gene

Ovarian cancer: risk factors relate to increased number of ovulations

Ovarian cancer

Ovarian cancer is the fifth most common malignancy in females. The peak age of incidence is 60 years and it generally carries a poor prognosis due to late diagnosis. Around 90% of ovarian cancers are epithelial in origin.

Risk factors
family history: mutations of the BRCA1 or the BRCA2 gene
many ovulations: early menarche, late menopause, nulliparity

It is traditionally taught that infertility treatment increases the risk of ovarian cancer, as it increases the number of ovulations. Recent evidence however suggests that there is not a significant link. The combined oral contraceptive pill reduces the risk (fewer ovulations) as does having many pregnancies.

Clinical features are notoriously vague
abdominal distension and bloating
abdominal and pelvic pain
urinary symptoms e.g. Urgency
early satiety
diarrhoea

Diagnosis is difficult and usually involves diagnostic laparotomy

How well did you know this?
1
Not at all
2
3
4
5
Perfectly
59
Q

A 57-year-old female presents due to problems with urine leakage over the past six months. She describes frequent voiding and not always being able to get to the toilet in time. She denies losing urine when coughing or sneezing. What is the most appropriate initial treatment?

Trial of oxybutynin
Bladder retraining
Regular toileting
Pelvic floor muscle training
Topical oestrogen cream
A

Urinary incontinence - first-line treatment:
urge incontinence: bladder retraining
stress incontinence: pelvic floor muscle training

Urinary incontinence

Urinary incontinence (UI) is a common problem, affecting around 4-5% of the population. It is more common in elderly females.

Causes
overactive bladder (OAB)/urge incontinence: due to detrusor over activity
stress incontinence: leaking small amounts when coughing or laughing
mixed incontinence: both urge and stress
overflow incontinence: due to bladder outlet obstruction, e.g. due to prostate enlargement

Initial investigation
bladder diaries should be completed for a minimum of 3 days
vaginal examination to exclude cystocele
urine dipstick and culture

Management depends on whether urge or stress UI is the predominant picture. If urge incontinence is predominant:
bladder retraining (lasts for a minimum of 6 weeks, the idea is to gradually increase the intervals between voiding)
bladder stabilising drugs:antimuscarinic) is first-line
surgical management: e.g. sacral nerve stimulation

If stress incontinence is predominant:
pelvic floor muscle training (for a minimum of 3 months)
surgical procedures: e.g. retropubic mid-urethral tape procedures

How well did you know this?
1
Not at all
2
3
4
5
Perfectly
60
Q

What is the main mechanism of action of Nexplanon (etonogestrel contraceptive implant)?

Thickens cervical mucus
Thins endometrial lining
Inhibition of ovulation
Causes fallopian tube dysfunction
Prevents implantation
A

Nexplanon - main mechanism of action is inhibition of ovulation

Implantable contraceptives

Implanon was a non-biodegradable subdermal contraceptive implant which has been replaced by Nexplanon. From a pharmacological perspective Nexplanon is the same as Implanon. The two main differences are:
the applicator has been redesigned to try and prevent ‘deep’ insertions (i.e. subcutaneous/intramuscular)
it is radiopaque and therefore easier to locate if impalpable

Both versions slowly releases the progestogen hormone etonogestrel. They are typically inserted in the proximal non-dominant arm, just overlying the tricep. The main mechanism of action is preventing ovulation. They also work by thickening the cervical mucus.

Key points
highly effective: failure rate 0.07/100 women/year
long-acting: lasts 3 years
doesn’t contain oestrogen so can be used if past history of thromboembolism, migraine etc
can be inserted immediately following a termination of pregnancy

Disadvantages include
the need for a trained professional to insert and remove device
additional contraceptive methods are needed for the first 7 days if not inserted on day 1 to 5 of a woman’s menstrual cycle

Adverse effects
irregular/heavy bleeding is the main problem
‘progestogen effects’: headache, nausea, breast pain

Interactions
enzyme-inducing drugs such as certain antiepileptic and rimampicin may reduce the efficacy of Nexplanon
the FSRH advises that women should be advised to switch to a method unaffected by enzyme-inducing drugs or to use additional contraception until 28 days after stopping the treatment

Contraindications
UKMEC 3*: ischaemic heart disease/stroke (for continuation, if initiation then UKMEC 2), unexplained, suspicious vaginal bleeding, past breast cancer, severe liver cirrhosis, liver cancer, positive antiphospholipid antibodies**
UKMEC 4**: current breast cancer

proven risks generally outweigh the advantages
**there is some contradiction in the guidance issued by the FSRH but their most recent document (revised 2010) lists positive antiphospholipid antibodies as UKMEC 3
**
a condition which represents an unacceptable risk if the contraceptive method is used

61
Q

A 29-year-old female who is 14 weeks into her first pregnancy is investigated for excessive sweating and tremor. Blood tests reveal the following:

TSH

A

Propylthiouracil is traditionally taught as the antithyroid drug of choice in pregnancy. This approach was supported by the 2007 Endocrine Society consensus guidelines. It also has the advantage of being excreted to a lesser extent than carbimazole in breast milk.

Despite this some endocrinologists use carbimazole and the BNF states both drugs may be used in pregnancy. Carbimazole has rarely been associated with aplasia cutis of the neonate

Pregnancy: thyroid problems

In pregnancy there is an increase in the levels of thyroxine-binding globulin (TBG). This causes an increase in the levels of total thyroxine but does not affect the free thyroxine level

Thyrotoxicosis

Untreated thyrotoxicosis increases the risk of fetal loss, maternal heart failure and premature labour

Graves’ disease is the most common cause of thyrotoxicosis in pregnancy. It is also recognised that activation of the TSH receptor by HCG may also occur - often termed transient gestational hyperthyroidism. HCG levels will fall in second and third trimester

Management
propylthiouracil has traditionally been the antithyroid drug of choice. This approach was supported by the 2007 Endocrine Society consensus guidelines
maternal free thyroxine levels should be kept in the upper third of the normal reference range to avoid fetal hypothyroidism
thyrotrophin receptor stimulating antibodies should be checked at 30-36 weeks gestation - helps to determine risk of neonatal thyroid problems
block-and-replace regimes should not be used in pregnancy
radioiodine therapy is contraindicated

Hypothyroidism

Key points
thyroxine is safe during pregnancy
serum thyroid stimulating hormone measured in each trimester and 6-8 weeks post-partum
some women require an increased dose of thyroxine during pregnancy
breast feeding is safe whilst on thyroxine

62
Q

Which one of the following is an absolute contraindication to the use of the progesterone only pill?

Immobility following surgery
Breast cancer 3 years ago
Previous stroke
History of antiphospholipid syndrome
Concurrent use of rifampicin
A

Breast cancer 3 years ago

Progestogen only pill: contraindications

The decision of whether to start a women a particular type of contraceptive is now guided by the UK Medical Eligibility Criteria (UKMEC). This scale categorises the potential cautions and contraindications according to a four point scale, as detailed below:
UKMEC 1: a condition for which there is no restriction for the use of the contraceptive method
UKMEC 2: advantages generally outweigh the disadvantages
UKMEC 3: disadvantages generally outweigh the advantages
UKMEC 4: represents an unacceptable health risk

Examples of UKMEC 3 conditions include
active liver disease or past tumour
liver enzyme inducers
breast cancer more than 5 years ago
undiagnosed vaginal bleeding

Examples of UKMEC 4 conditions include
pregnancy
breast cancer within the last 5 years

63
Q

Which one of the following statements regarding gestational trophoblastic disorders is incorrect?

In a complete hydatidiform mole all 46 chromosomes are of paternal origin
Very high serum levels of human chorionic gonadotropin (hCG) are seen
Symptoms of hyperthyroidism may be seen with a complete hydatidiform mole
Less than 1% of women with a complete hydatidiform mole go on to develop choriocarcinoma
Effective contraception is recommended to avoid pregnancy in the next 12 months
A

Around 2-3% go on to develop choriocarcinoma

Gestational trophoblastic disorders

Describes a spectrum of disorders originating from the placental trophoblast:
complete hydatidiform mole
partial hydatidiform mole
choriocarcinoma

Complete hydatidiform mole

Benign tumour of trophoblastic material. Occurs when an empty egg is fertilized by a single sperm that then duplicates its own DNA, hence the all 46 chromosomes are of paternal origin

Features
bleeding in first or early second trimester
exaggerated symptoms of pregnancy e.g. hyperemesis
uterus large for dates
very high serum levels of human chorionic gonadotropin (hCG)
hypertension and hyperthyroidism* may be seen

Management
urgent referral to specialist centre - evacuation of the uterus is performed
effective contraception is recommended to avoid pregnancy in the next 12 months

Around 2-3% go on to develop choriocarcinoma

In a partial mole a normal haploid egg may be fertilized by two sperms, or by one sperm with duplication of the paternal chromosomes. Therefore the DNA is both maternal and paternal in origin. Usually triploid - e.g. 69 XXX or 69 XXY. Fetal parts may be seen

*hCG can mimic thyroid-stimulating hormone (TSH)

64
Q

Which one of the following statements regarding endometriosis is incorrect?

Laparoscopy is the gold-standard investigation
Up to 10-15% of women have a degree of endometriosis
Is a cause of secondary dysmenorrhoea
Is a cause of deep dyspareunia
Laparoscopic findings correlate well with the severity of symptoms and can guide treatment
A

Laparoscopic findings correlate well with the severity of symptoms and can guide treatment

Endometriosis

Endometriosis is a common condition characterised by the growth of ectopic endometrial tissue outside of the uterine cavity. Up to 10-15% of women have a degree of endometriosis

Clinical features
chronic pelvic pain
dysmenorrhoea - pain often starts days before bleeding
deep dyspareunia
subfertility

Less common features
urinary symptoms e.g. dysuria, urgency
dyschezia (painful bowel movements)

Investigation
laparoscopy is the gold-standard investigation
there is little role for investigation in primary care (e.g. ultrasound)- if the symptoms are significant the patient should be referred for a definitive diagnosis

Management depends on clinical features - there is poor correlation between laparoscopic findings and severity of symptoms
NSAIDs and other analgesia for symptomatic relief
combined oral contraceptive pill
progestogens e.g. medroxyprogesterone acetate
gonadotrophin-releasing hormone (GnRH) analogues - said to induce a ‘pseudomenopause’ due to the low oestrogen levels
intrauterine system (Mirena)
drug therapy unfortunately does not seem to have a significant impact on fertility rates

Surgery
some treatments such as laparoscopic excision and laser treatment of endometriotic ovarian cysts may improve fertility

65
Q

Which one of the following is most associated with combined oral contraceptive pill use?

Increased dysmenorrhoea
Increased incidence of benign breast disease
Worsening of acne
Increased risk of colorectal cancer
Increased risk of cervical cancer
A

Combined oral contraceptive pill
increased risk of breast and cervical cancer
protective against ovarian and endometrial cancer

Combined oral contraceptive pill: advantages/disadvantages

Advantages of combined oral contraceptive pill
highly effective (failure rate

66
Q

In which one of the following situations would you not routinely give anti-D prophylaxis to a non-sensitised Rhesus negative mother?

Amniocentesis at 16 weeks
External cephalic version at 37 weeks
Spontaneous miscarriage at 10 weeks
Placental abruption at 26 weeks
Ectopic pregnancy at 7 weeks
A

Spontaneous miscarriage at 10 weeks

Rhesus negative pregnancy

A basic understanding of the pathophysiology is essential to understand the management of Rhesus negative pregnancies
along with the ABO system the Rhesus system is the most important antigen found on red blood cells. The D antigen is the most important antigen of the rhesus system
around 15% of mothers are rhesus negative (Rh -ve)
if a Rh -ve mother delivers a Rh +ve child a leak of fetal red blood cells may occur
this causes anti-D IgG antibodies to form in mother
in later pregnancies these can cross placenta and cause haemolysis in fetus
this can also occur in the first pregnancy due to leaks

Prevention
test for D antibodies in all Rh -ve mothers at booking
NICE (2008) advise giving anti-D to non-sensitised Rh -ve mothers at 28 and 34 weeks
the evidence base suggests that there is little difference in the efficacy of single-dose (at 28 weeks) and double-dose regimes (at 28 & 34 weeks). For this reason the RCOG in 2011 advised that either regime could be used ‘depending on local factors’
anti-D is prophylaxis - once sensitization has occurred it is irreversible
if event is in 2nd/3rd trimester give large dose of anti-D and perform Kleihauer test - determines proportion of fetal RBCs present

Anti-D immunoglobulin should be given as soon as possible (but always within 72 hours) in the following situations:
delivery of a Rh +ve infant, whether live or stillborn
any termination of pregnancy
miscarriage if gestation is > 12 weeks
ectopic pregnancy
external cephalic version
antepartum haemorrhage
amniocentesis, chorionic villus sampling, fetal blood sampling

Tests
all babies born to Rh -ve mother should have cord blood taken at delivery for FBC, blood group & direct Coombs test
Coombs test: direct antiglobulin, will demonstrate antibodies on RBCs of baby
Kleihauer test: add acid to maternal blood, fetal cells are resistant

Affected fetus
oedematous (hydrops fetalis, as liver devoted to RBC production albumin falls)
jaundice, anaemia, hepatosplenomegaly
heart failure
kernicterus
treatment: transfusions, UV phototherapy

67
Q

A pregnant woman in her second trimester asks for advice about malaria prophylaxis. Which one of the following statements is true?

Doxycycline can be used in the third trimester
Chloroquine should be avoided in the first and second trimesters
Mefloquine is the anti-malarial of choice in pregnancy
Malarone (atovaquone + proguanil) is safe in all trimesters of pregnancy
Folic acid should be prescribed for pregnant patients taking proguanil
A

Folic acid should be prescribed for pregnant patients taking proguanil

Malaria: prophylaxis

There are around 1,500-2,000 cases each year of malaria in patients returning from endemic countries. The majority of these cases (around 75%) are caused by the potentially fatal Plasmodium falciparum protozoa. The majority of patients who develop malaria did not take prophylaxis. It should also be remembered that UK citizens who originate from malaria endemic areas quickly lose their innate immunity.

Pregnant women should be advised to avoid travelling to regions where malaria is endemic. Diagnosis can also be difficult as parasites may not be detectable in the blood film due to placental sequestration. However, if travel cannot be avoided:

chloroquine can be taken
proguanil: folate supplementation (5mg od) should be given
Malarone (atovaquone + proguanil): the BNF advises to avoid these drugs unless essential. If taken then folate supplementation should be given
mefloquine: caution advised
doxycycline is contraindicated

It is again advisable to avoid travel to malaria endemic regions with children if avoidable. However, if travel is essential then children should take malarial prophylaxis as they are more at risk of serious complications.
diethyltoluamide (DEET) 20-50% can be used in children over 2 months of age
doxycycline is only licensed in the UK for children over the age of 12 years

68
Q

Which one of the following is an absolute contraindication to combined oral contraceptive pill use?

Concurrent use of St John's Wort
Family history of thromboembolic disease in first degree relatives
A

Migraine with aura

Combined oral contraceptive pill: contraindications

The decision of whether to start a women on the combined oral contraceptive pill is now guided by the UK Medical Eligibility Criteria (UKMEC). This scale categorises the potential cautions and contraindications according to a four point scale, as detailed below:
UKMEC 1: a condition for which there is no restriction for the use of the contraceptive method
UKMEC 2: advantages generally outweigh the disadvantages
UKMEC 3: disadvantages generally outweigh the advantages
UKMEC 4: represents an unacceptable health risk

Examples of UKMEC 3 conditions include
more than 35 years old and smoking less than 15 cigarettes/day
BMI > 35 kg/m^2*
migraine without aura and more than 35 years old
family history of thromboembolic disease in first degree relatives 20 years ago is classified as UKMEC 3 or 4 depending on severity

*The UKMEC 4 rating for a BMI > 40 kg/m^2 was removed in 2009.

69
Q

Which one of the following statements regarding dysmenorrhoea is correct?

Around 10% of women experience dysmenorrhoea
The pain of secondary dysmenorrhoea typically develops 3-4 days before the onset of the period
The intrauterine system is the first-line treatment for secondary dysmenorrhoea
Primary dysmenorrhoea typically develops around 10 years after the menarche
Endometriosis is a cause of primary dysmenorrhoea
A

The pain of secondary dysmenorrhoea develops 3-4 days before the onset of the period

The treatment of secondary dysmenorrhoea depends on the underlying cause.

Dysmenorrhoea

Dysmenorrhoea is characterised by excessive pain during the menstrual period. It is traditionally divided into primary and secondary dysmenorrhoea.

Primary dysmenorrhoea

In primary dysmenorrhoea there is no underlying pelvic pathology. It affects up to 50% of menstruating women and usually appears within 1-2 years of the menarche. Excessive endometrial prostaglandin production is thought to be partially responsible.

Features
pain typically starts just before or within a few hours of the period starting
suprapubic cramping pains which may radiate to the back or down the thigh

Management
NSAIDs such as mefenamic acid and ibuprofen are effective in up to 80% of women. They work by inhibiting prostaglandin production
combined oral contraceptive pills are used second line

Secondary dysmenorrhoea

Secondary dysmenorrhoea typically develops many years after the menarche and is the result of an underlying pathology. In contrast to primary dysmenorrhoea the pain usually starts 3-4 days before the onset of the period. Causes include:
endometriosis
adenomyosis
pelvic inflammatory disease
intrauterine devices*
fibroids

Clinical Knowledge Summaries recommend referring all patients with secondary dysmenorrhoea to gynaecology for investigation.

*this refers to normal copper coils. Note that the intrauterine system (Mirena) may help dysmenorrhoea

70
Q

Which one of the following statements regarding cervical cancer screening is incorrect?

Women aged 25-49 should be offered 3-yearly screening
The optimum time to take a cervical smear is around mid-cycle
Detection of cervical adenocarcinomas has significantly improved since the introduction of liquid based cytology
Liquid based cytology reduces the rate of inadequate smears
Liquid based cytology has an increased sensitivity and specificity
A

Cervical cancer screening detects squamous cell cancer and may miss adenocarcinomas

The change to liquid based cytology has unfortunately not improved the detection rate of adenocarcinomas

Cervical cancer screening

The UK has a well established cervical cancer screening program which is estimated to prevent 1,000-4,000 deaths per year. It should be noted that cervical adenocarcinomas, which account for around 15% of cases, are frequently undetected by screening

Who is screened and how often?

A smear test is offered to all women between the ages of 25-64 years
25-49 years: 3-yearly screening
50-64 years: 5-yearly screening

How is performed?

There is currently a move away from traditional Papanicolaou (Pap) smears to liquid-based cytology (LBC). Rather than smearing the sample onto a slide the sample is either rinsed into the preservative fluid or the brush head is simply removed into the sample bottle containing the preservative fluid.

Advantages of LBC includes
reduced rate of inadequate smears
increased sensitivity and specificity

It is said that the best time to take a cervical smear is around mid-cycle. Whilst there is limited evidence to support this it is still the current advice given out by the NHS.

In Scotland women from the ages of 20-60 years are screened every 3 years.

71
Q

A female patient asks for advice about having an intrauterine device inserted (a TT380 Slimline). What advice should be given regarding the likely effect on her periods?

Periods will tend to be longer, heavier and more painful
They will stop after 6 months in > 50% of users
Periods will tend to be lighter, shorter and less painful
Continual, light bleeding is seen in 50%
They will stop after 6 months in > 90% of users
A

Periods will tend to be longer, heavier and more painful

Intrauterine contraceptive devices

Intrauterine contraceptive devices comprise both conventional copper intrauterine devices (IUDs) and levonorgestrel-releasing intrauterine systems (IUS, Mirena). The IUS is also used in the management of menorrhagia

Effectiveness
both the IUD and IUS are more than 99% effective

Mode of action
IUD: primary mode of action is prevention of fertilisation by causing decreased sperm motility and survival (possibly an effect of copper ions)
IUS: levonorgestrel prevents endometrial proliferation and causes cervical mucous thickening

Counselling
IUD is effective immediately following insertion
IUS can be relied upon after 7 days

Potential problems
IUDs make periods heavier, longer and more painful
the IUS is associated with initial frequent uterine bleeding and spotting. Later women typically have intermittent light menses with less dysmenorrhoea and some women become amenorrhoeic
uterine perforation: up to 2 per 1000 insertions
the proportion of pregnancies that are ectopic is increased but the absolute number of ectopic pregnancies is reduced, compared to a woman not using contraception
infection: there is a small increased risk of pelvic inflammatory disease in the first 20 days after insertion but after this period the risk returns to that of a standard population
expulsion: risk is around 1 in 20, and is most likely to occur in the first 3 months

72
Q

A 24-year-old female has an abdominal ultrasound performed as she has had repeat urinary tract infections in the past 12 months. It is reported as follows:

Both kidneys are normal size. No abnormality of the urinary tract is noted.

Liver, spleen, pancreas are normal

Right ovary and uterus normal.

4cm simple ovarian cyst noted on left ovary

End of report.

What is the most appropriate action?

Repeat ultrasound in 12 weeks
Start a combined oral contraceptive pill
Urgent referral to gynaecology
Check CA125
Routine referral to gynaecology
A

Repeat ultrasound in 12 weeks

If the cyst persists after 12 weeks referral should be considered.

Ovarian enlargement: management

The initial imaging modality for suspected ovarian cysts/tumours is ultrasound. The report will usually report that the cyst is either:
simple: unilocular, more likely to be physiological or benign
complex: multilocular, more likely to be malignant

Management depends on the age of the patient and whether the patient is symptomatic. It should be remembered that the diagnosis of ovarian cancer is often delayed due to a vague presentation.

Premenopausal women
a conservative approach may be taken for younger women (especially if

73
Q

What is the main mechanism of action of Cerazette (desogestrel)?

Thickens cervical mucous
Inhibits ovulation
Causes endometrial thinning
Inhibits implantation
Toxic to sperm
A

Cerazette inhibits ovulation in around 97-99% of cycles.

Progestogen only pill: types

Second generation
norethisterone
levonorgestrel
ethynodiol diacetate

Third generation
desogestrel (Cerazette)

Cerazette
new third generation type of progestogen only pill (POP) containing desogestrel
inhibits ovulation in the majority of women
users can take the pill up to 12 hours late rather than 3 hours like other POPs

74
Q

An obese 48-year-old woman presents for advice regarding contraception. She has recently met a new partner but is sure she does not want any children. She states that her periods have become heavier recently and she describes some intermenstrual bleeding. What is the most suitable management?

Suggest she considers female sterilisation
Suggest her partner comes in to discuss vasectomy
Refer to gynaecology
Suggest she considers a intrauterine system (Mirena coil)
Check FSH levels
A

Given the history of intermenstrual bleeding this patient should be referred to gynaecology to exclude endometrial cancer

Endometrial cancer

Endometrial cancer is classically seen in post-menopausal women but around 25% of cases occur before the menopause. It usually carries a good prognosis due to early detection

The risk factors for endometrial cancer are as follows*:
obesity
nulliparity
early menarche
late menopause
unopposed oestrogen. The addition of a progestogen to oestrogen reduces this risk (e.g. In HRT). The BNF states that the additional risk is eliminated if a progestogen is given continuously
diabetes mellitus
tamoxifen
polycystic ovarian syndrome

Features
post-menopausal bleeding is the classic symptom
pre-menopausal women may have a change intermenstrual bleeding
pain and discharge are unusual features

Investigation
first-line investigation is trans-vaginal ultrasound - a normal endometrial thickness (

75
Q

A 31-year-old female with a history of epilepsy consults you following an uneventful pregnancy. Which one of the following drugs would it be safe to continue during breast feeding?

Phenytoin
Carbamazepine
Lamotrigine
Sodium valproate
All of the above
A

Breast feeding is acceptable with nearly all anti-epileptic drugs

The BNF states ‘breast-feeding is acceptable with all antiepileptic drugs, taken in normal doses, with the possible exception of barbiturates’

Epilepsy: pregnancy and breast feeding

The risks of uncontrolled epilepsy during pregnancy generally outweigh the risks of medication to the fetus. All women thinking about becoming pregnant should be advised to take folic acid 5mg per day well before pregnancy to minimise the risk of neural tube defects. Around 1-2% of newborns born to non-epileptic mothers have congenital defects. This rises to 3-4% if the mother takes antiepileptic medication.

Other points
aim for monotherapy
there is no indication to monitor antiepileptic drug levels
sodium valproate: associated with neural tube defects
carbamazepine: often considered the least teratogenic of the older antiepileptics
phenytoin: associated with cleft palate
lamotrigine: studies to date suggest the rate of congenital malformations may be low. The dose of lamotrigine may need to be increased in pregnancy

Breast feeding is generally considered safe for mothers taking antiepileptics with the possible exception of the barbiturates

It is advised that pregnant women taking phenytoin are given vitamin K in the last month of pregnancy to prevent clotting disorders in the newborn

Sodium valproate

The November 2013 issue of the Drug Safety Update also carried a warning about new evidence showing a significant risk of neurodevelopmental delay in children following maternal use of sodium valproate.

The update concludes that sodium valproate should not be used during pregnancy and in women of childbearing age unless clearly necessary. Women of childbearing age should not start treatment without specialist neurological or psychiatric advice.

76
Q

A 23-year-old woman who is 10 weeks pregnant presents with severe vomiting. She is now having difficulty keeping down fluids and a dipstick of her urine shows ketones ++. Which one of the following is not associated with an increased risk of this condition?

Obesity
Trophoblastic disease
Smoking
Nulliparity
Carrying twins
A

Smoking is associated with a decreased incidence of hyperemesis gravidarum

Hyperemesis gravidarum

Hyperemesis gravidarum describes excessive vomiting during pregnancy. It occurs in around 1% of pregnancies and is thought to be related to raised beta hCG levels. Hyperemesis gravidarum is most common between 8 and 12 weeks but may persist up to 20 weeks*.

Associations
multiple pregnancies
trophoblastic disease
hyperthyroidism
nulliparity
obesity

Smoking is associated with a decreased incidence of hyperemesis

Management
antihistamines should be used first-line (BNF suggests promethazine as first-line)
admission may be needed for IV hydration

Complications
Wernicke’s encephalopathy
Mallory-Weiss tear
central pontine myelinolysis
acute tubular necrosis
fetal: small for gestational age, pre-term birth

*and in very rare cases beyond 20 weeks

77
Q

A 25-year-old woman with primary antiphospholipid syndrome is reviewed. She has just had a booking ultrasound at 11 weeks gestation which confirms a viable pregnancy. This is her first pregnancy and she is otherwise fit and well. Which one of the following is the recommend treatment?

Aspirin + prednisolone
Low-molecular weight heparin
Prednisolone + low-molecular weight heparin
Aspirin + low-molecular weight heparin
Aspirin
A

Antiphospholipid syndrome in pregnancy: aspirin + LMWH

The ultrasound at 11 weeks gestation would show a fetal heart if the pregnancy was viable. This patient should therefore be taking both aspirin and low-molecular weight heparin.

Antiphospholipid syndrome: pregnancy

Antiphospholipid syndrome is an acquired disorder characterised by a predisposition to both venous and arterial thromboses, recurrent fetal loss and thrombocytopenia. It may occur as a primary disorder or secondary to other conditions, most commonly systemic lupus erythematosus (SLE)

In pregnancy the following complications may occur:
recurrent miscarriage
IUGR
pre-eclampsia
placental abruption
pre-term delivery
venous thromboembolism

Management
low-dose aspirin should be commenced once the pregnancy is confirmed on urine testing
low molecular weight heparin once a fetal heart is seen on ultrasound. This is usually discontinued at 34 weeks gestation
these interventions increase the live birth rate seven-fold

78
Q

A 24-year-old woman presents due to an itchy vulva and pain during sex. She also mentions a green, offensive vaginal discharge for the past 2 weeks. What is the most likely diagnosis?

Candida
Bacterial vaginosis
Gonorrhoea
Trichomonas vaginalis
Chlamydia
A

Trichomonas vaginalis

Vaginal discharge

Vaginal discharge is a common presenting symptom and is not always pathological

Common causes
physiological
Candida
Trichomonas vaginalis
bacterial vaginosis

Less common causes
whilst cervical infections such as Chlamydia and Gonorrhoea can cause a vaginal discharge this is rarely the presenting symptoms
ectropion
foreign body
cervical cancer

79
Q

You are reviewing the cervical screening results for a 31-year-old woman. Her previous smears have been normal. The results show mild dyskaryosis. The sample is then sent for HPV testing which is positive. What will happen next?

Offered colposcopy
Offered a course of Gardasil
Smear repeated in 6 months
Smear repeated in 1 year
Smear repeated in 3 years
A

If a smear is reported as borderline or mild dyskaryosis the original sample is tested for HPV
if HPV negative the patient goes back to routine recall
if HPV positive the patient is referred for colposcopy

80
Q

What is the youngest age that a child in the UK may be ‘legally able to consent to sexual activity’?

12
13
14
15
16
A

13

The Faculty for Sexual and Reproductive Health state: In England, Wales and Northern Ireland, those under the age of 13 years are considered unable to legally consent to sexual activity. In Northern Ireland, there is no statutory duty under criminal law to report to the police cases of sexual activity involving children under the age of 16 years unless the child is under 13 years or the other party is aged 18 years or over. When the Sexual Offences Scotland Act 2009 comes into force, sexual activity with a male or female aged under 13 years will be ‘rape of a young child’.

Contraceptive Choices for Young People

The Faculty of Sexual and Reproductive Health (FRSH) produced guidelines in 2010 concerning the provision of contraception to young people. Much of the following is based on those guidelines. Please see the link for more details.

Legal and ethical issues
the age of consent for sexual activity in the UK is 16 years. Practitioners may however provide advice and contraception if they feel that the young person is ‘competent’. This is usually assessed using the Fraser guidelines (see below)
children under the age of 13 years are considered unable to consent for sexual intercourse and hence consultations regarding this age group should automatically trigger child protection measures

The Fraser Guidelines state that all the following requirements should be fulfilled:
the young person understands the professional’s advice
the young person cannot be persuaded to inform their parents
the young person is likely to begin, or to continue having, sexual intercourse with or without contraceptive treatment
unless the young person receives contraceptive treatment, their physical or mental health, or both, are likely to suffer
the young person’s best interests require them to receive contraceptive advice or treatment with or without parental consent

Sexual Transmitted Infections (STIs)
young people should be advised to have STI tests 2 and 12 weeks after an incident of unprotected sexual intercourse (UPSI)

Choice of contraceptive
clearly long-acting reversible contraceptive methods (LARCs) have advantages in young people as this age group may often be less reliable in remembering to take medication
however, there are some concerns about the effect of progesterone-only injections (Depo-provera) on bone mineral density and the UKMEC category of the IUS and IUD is 2 for women under the age of 20 years, meaning they may not be the best choice
the progesterone-only implant (Nexplanon) is therefore the LARC of choice is young people

81
Q

A 31-year-old woman who is known to be HIV positive presents following a positive pregnancy test. Her last menstrual period was 6 weeks ago. The last CD4 count was 420 * 106/l and she does not take any antiretroviral therapy. What is the most appropriate management with regards to antiretroviral therapy?

Check CD4 at 12 weeks and initiate antiretroviral therapy if CD4 count is less than 350 * 106/l
Do not give antiretroviral therapy
Start antiretroviral therapy at 20-32 weeks
Start antiretroviral therapy at 10-12 weeks
Start antiretroviral therapy immediately
A

Whilst the RCOG guidelines recommend 28-32 weeks the BHIVA suggest a slightly earlier gestation may be suitable depending on individual circumstances. The most suitable answer is therefore 20-32 weeks.

HIV and pregnancy

With the increased incidence of HIV infection amongst the heterosexual population there are an increasing number of HIV positive women giving birth in the UK. In London the incidence may be as high as 0.4% of pregnant women. The aim of treating HIV positive women during pregnancy is to minimise harm to both the mother and fetus, and to reduce the chance of vertical transmission.

Guidelines regularly change on this subject and most recent guidelines can be found using the links provided.

Factors which reduce vertical transmission (from 25-30% to 2%)
maternal antiretroviral therapy
mode of delivery (caesarean section)
neonatal antiretroviral therapy
infant feeding (bottle feeding)

Screening
NICE guidelines recommend offering HIV screening to all pregnant women

Antiretroviral therapy
all pregnant women should be offered antiretroviral therapy regardless of whether they were taking it previously
if women are not currently taking antiretroviral therapy the RCOG recommend that it is commenced between 28 and 32 weeks of gestation and should be continued intrapartum. BHIVA recommend that antiretroviral therapy may be started at an earlier gestation depending upon the individual situation

Mode of delivery
vaginal delivery is recommenced if viral load is less than 50 copies/ml at 36 weeks, otherwise caesarian section is recommended
a zidovudine infusion should be started four hours before beginning the caesarean section

Neonatal antiretroviral therapy
zidovudine is usually administered orally to the neonate if maternal viral load is

82
Q

Which one of the following statements regarding pelvic inflammatory disease is incorrect?

Increases the risk of future ectopic pregnancy
Intrauterine contraceptive devices should always be removed following diagnosis
Deep dyspareunia is a typical feature
A low threshold for treatment is recommended
Chlamydia trachomatis is the most common cause
A

Intrauterine contraceptive devices do not need to be removed in mild cases of pelvic inflammatory disease

Pelvic inflammatory disease

Pelvic inflammatory disease (PID) is a term used to describe infection and inflammation of the female pelvic organs including the uterus, fallopian tubes, ovaries and the surrounding peritoneum. It is usually the result of ascending infection from the endocervix

Causative organisms
Chlamydia trachomatis - the most common cause
Neisseria gonorrhoeae
Mycoplasma genitalium
Mycoplasma hominis

Features
lower abdominal pain
fever
deep dyspareunia
dysuria and menstrual irregularities may occur
vaginal or cervical discharge
cervical excitation

Investigation
screen for Chlamydia and Gonorrhoea

Management
due to the difficulty in making an accurate diagnosis, and the potential complications of untreated PID, consensus guidelines recommend having a low threshold for treatment
oral ofloxacin + oral metronidazole or intramuscular ceftriaxone + oral doxycycline + oral metronidazole
RCOG guidelines suggest that in mild cases of PID intrauterine contraceptive devices may be left in. The more recent BASHH guidelines suggest that the evidence is limited but that ‘ Removal of the IUD should be considered and may be associated with better short term clinical outcomes’

Complications
infertility - the risk may be as high as 10-20% after a single episode
chronic pelvic pain
ectopic pregnancy

83
Q

Which one of the following is the best definition of secondary amenorrhoea?

Cessation of what a patient considers her normal periods for greater than 6 months
Cessation of established, regular menstruation for 6 months or longer
Cessation of what a patient considers her normal periods for greater than 12 months
Failure to have regular periods by the age of 16 years
Cessation of established, regular menstruation for 12 months or longer
A

Cessation of established, regular menstruation for 6 months or longer

Amenorrhoea

Amenorrhoea may be divided into primary (failure to start menses by the age of 16 years) or secondary (cessation of established, regular menstruation for 6 months or longer).

Causes of primary amenorrhoea
Turner’s syndrome
testicular feminisation
congenital adrenal hyperplasia
congenital malformations of the genital tract

Causes of secondary amenorrhoea (after excluding pregnancy)
hypothalamic amenorrhoea (e.g. Stress, excessive exercise)
polycystic ovarian syndrome (PCOS)
hyperprolactinaemia
premature ovarian failure
thyrotoxicosis*

Initial investigations
exclude pregnancy with urinary or serum bHCG
gonadotrophins: low levels indicate a hypothalamic cause where as raised levels suggest an ovarian problem (e.g. Premature ovarian failure)
prolactin
androgen levels: raised levels may be seen in PCOS
oestradiol
thyroid function tests

*hypothyroidism may also cause amenorrhoea

84
Q

A pregnant woman presents for review. She is 24 weeks pregnant. What would be the expected symphysis-fundal height?

13 - 15 cm
15 - 17 cm
17 - 19 cm
18 - 22 cm
22 - 26 cm
A

After 20 weeks, symphysis-fundal height in cm = gestation in weeks

Symphysis-fundal height

The symphysis-fundal height (SFH) is measured from the top of the pubic bone to the top of the uterus in centimetres

It should match the gestational age in weeks to within 2 cm after 20 weeks, e.g. if 24 weeks then the a normal SFH = 22 to 26 cm

85
Q

Which one of the following statements regarding breech presentation is incorrect?

External cephalic version should be considered at 36 weeks in primips
Maternal obesity is a risk factor
Vaginal delivery is an option for delivery
Describes the caudal end of the fetus occupying the lower segment
Occurs in around 3% of babies near term
A

Breech presentation risk factors - uterine malformations, placenta praevia, polyhydramnios or oligohydramnios. NOT maternal obesity

Breech presentation

In a breech presentation the caudal end of the fetus occupies the lower segment. Whilst around 25% of pregnancies at 28 weeks are breech it only occurs in 3% of babies near term. A frank breech is the most common presentation with the hips flexed and knees fully extended. A footling breech, where one or both feet come first with the bottom at a higher position, is rare but carries a higher perinatal morbidity

Risk factors for breech presentation
uterine malformations, fibroids
placenta praevia
polyhydramnios or oligohydramnios
fetal abnormality (e.g. CNS malformation, chromosomal disorders)
prematurity (due to increased incidence earlier in gestation)

Cord prolapse is more common in breech presentations

Management
if

86
Q

Which one of the following regarding the management of thyroid problems during pregnancy is incorrect?

Maternal free thyroxine levels should be kept in the upper third of the normal reference range when treating thyrotoxicosis
Increased levels of thyroxine-binding globulin are seen in pregnancy
Block-and-replace is preferable in pregnancy compared to antithyroid drug titration
Breast feeding is safe whilst on thyroxine
Untreated thyrotoxicosis increases the risk of premature labour
A

Block-and-replace is preferable in pregnancy compared to antithyroid drug titration

Pregnancy: thyroid problems

In pregnancy there is an increase in the levels of thyroxine-binding globulin (TBG). This causes an increase in the levels of total thyroxine but does not affect the free thyroxine level

Thyrotoxicosis

Untreated thyrotoxicosis increases the risk of fetal loss, maternal heart failure and premature labour

Graves’ disease is the most common cause of thyrotoxicosis in pregnancy. It is also recognised that activation of the TSH receptor by HCG may also occur - often termed transient gestational hyperthyroidism. HCG levels will fall in second and third trimester

Management
propylthiouracil has traditionally been the antithyroid drug of choice. This approach was supported by the 2007 Endocrine Society consensus guidelines
maternal free thyroxine levels should be kept in the upper third of the normal reference range to avoid fetal hypothyroidism
thyrotrophin receptor stimulating antibodies should be checked at 30-36 weeks gestation - helps to determine risk of neonatal thyroid problems
block-and-replace regimes should not be used in pregnancy
radioiodine therapy is contraindicated

Hypothyroidism

Key points
thyroxine is safe during pregnancy
serum thyroid stimulating hormone measured in each trimester and 6-8 weeks post-partum
some women require an increased dose of thyroxine during pregnancy
breast feeding is safe whilst on thyroxine

87
Q

Theme: Pelvic pain

A. Appendicitis
B. Ovarian torsion
C. Urogenital prolapse
D. Endometriosis
E. Diverticular disease
F. Ovarian cyst
G. Adenomyosis
H. Irritable bowel syndrome
I. Pelvic inflammatory disease
J. Ectopic pregnancy

For each one of the following scenarios please select the most likely diagnosis:

A 33-year-old woman is investigated for infertility. Laparoscopy is essentially normal. Hysterosalpingography shows blocked fallopian tubes bilaterally.

A

Pelvic inflammatory disease

Endometriosis would normally be seen during laparoscopy.

88
Q

Theme: Pelvic pain

A. Appendicitis
B. Ovarian torsion
C. Urogenital prolapse
D. Endometriosis
E. Diverticular disease
F. Ovarian cyst
G. Adenomyosis
H. Irritable bowel syndrome
I. Pelvic inflammatory disease
J. Ectopic pregnancy

For each one of the following scenarios please select the most likely diagnosis:

A 27-year-old woman complains of spasmodic pains in the left iliac fossa. These pains have been present for the past six months and sometimes radiate to the back. She often feels bloated, particularly around her period. She describes her bowels as being ‘stubborn’ but does not take a regular laxative. Vaginal and abdominal examination is unremarkable.

A

Irritable bowel syndrome

89
Q

Theme: Pelvic pain

A. Appendicitis
B. Ovarian torsion
C. Urogenital prolapse
D. Endometriosis
E. Diverticular disease
F. Ovarian cyst
G. Adenomyosis
H. Irritable bowel syndrome
I. Pelvic inflammatory disease
J. Ectopic pregnancy

For each one of the following scenarios please select the most likely diagnosis:

A 26-year-old women develops sudden onset right iliac fossa pains whilst playing netball. She is nauseous and has vomited twice. On examination she is tender in the right iliac fossa.

A

Ovarian torsion

The pain of appendicitis is unlikely to be of sudden onset.

90
Q

Which one of the following is not a risk factor for the development of pre-eclampsia?

Previous history of pre-eclampsia
Body mass index of 34 kg/m^2
Age of 42 years
Multiple pregnancy
Multiparity
A

No previous pregnancies is a risk factor for pre-eclampsia

Pre-eclampsia

Pre-eclampsia is a condition seen after 20 weeks gestation characterised by pregnancy-induced hypertension in association with proteinuria (> 0.3g / 24 hours). Oedema used to be third element of the classic triad but is now often not included in the definition as it is not specific

Pre-eclampsia is important as it predisposes to the following problems
fetal: prematurity, intrauterine growth retardation
eclampsia
haemorrhage: placental abruption, intra-abdominal, intra-cerebral
cardiac failure
multi-organ failure

Risk factors
> 40 years old
nulliparity (or new partner)
multiple pregnancy
body mass index > 30 kg/m^2
diabetes mellitus
pregnancy interval of more than 10 years
family history of pre-eclampsia
previous history of pre-eclampsia
pre-existing vascular disease such as hypertension or renal disease

Features of severe pre-eclampsia
hypertension: typically > 170/110 mmHg and proteinuria as above
proteinuria: dipstick ++/+++
headache
visual disturbance
papilloedema
RUQ/epigastric pain
hyperreflexia
platelet count 160/110 mmHg although many clinicians have a lower threshold
oral labetalol is now first-line following the 2010 NICE guidelines. Nifedipine and hydralazine may also be used
delivery of the baby is the most important and definitive management step. The timing depends on the individual clinical scenario

91
Q

A 30-year-old woman who is 38 weeks pregnant presents with dysuria and urinary frequency. A urine dipstick is positive for nitrites and leucocytes. Of the options given, what is the most suitable management?

Ciprofloxacin
Cefalexin
Advise to increase fluid intake and take cranberry juice.
Doxycycline
Nitrofurantoin
A

Cafalexin

Amoxicillin is also recommended in this situation. Nitrofurantoin should be avoided near term as it may cause neonatal haemolysis but it may be used earlier in the pregnancy.

This lady is highly likely to have a urinary tract infection so advising her just to use cranberry juice is inappropriate.

Urinary tract infection in adults: management

Lower urinary tract infections in non-pregnant women
local antibiotic guidelines should be followed if available
2012 SIGN guidelines recommend trimethoprim or nitrofurantoin for 3 days

Pregnant women with symptomatic bacteriuria should be treated with an antibiotic for 7 days. A urine culture should be sent. For asymptomatic pregnant women:
a urine culture should be performed routinely at the first antenatal visit
if positive, a second urine culture should be sent to confirm the presence of bacteriuria
SIGN recommend to treat asymptomatic bacteriuria detected during pregnancy with an antibiotic
a 7 day course of antibiotics should be given
a further urine culture should be sent following completion of treatment as a test of cure

For patients with sign of acute pyelonephritis hospital admission should be considered
local antibiotic guidelines should be followed if available
the BNF currently recommends a broad-spectrum cephalosporin or a quinolone for 10-14 days

92
Q

A 22-year-old female in the second trimester of pregnancy presents with an offensive vaginal discharge. History and examination findings are consistent with a diagnosis of bacterial vaginosis. What is the most appropriate initial management?

Advise risks of treatments outweigh benefits in pregnancy
Topical clindamycin
Oral metronidazole
Clotrimazole pessary
Advice regarding hygiene and cotton underwear
A

Bacterial vaginosis in pregnancy: still use oral metronidazole

Bacterial vaginosis is a risk factor for a number of pregnancy related problems including preterm labour. It was traditionally taught to avoid oral metronidazole in the first trimester but guidelines now recommend its use throughout pregnancy. Please see the Clinical Knowledge Summary link

Bacterial vaginosis

Bacterial vaginosis (BV) describes an overgrowth of predominately anaerobic organisms such as Gardnerella vaginalis. This leads to a consequent fall in lactic acid producing aerobic lactobacilli resulting in a raised vaginal pH.

Whilst BV is not a sexually transmitted infection it is seen almost exclusively in sexually active women.

Features
vaginal discharge: ‘fishy’, offensive
asymptomatic in 50%

Amsel’s criteria for diagnosis of BV - 3 of the following 4 points should be present
thin, white homogenous discharge
clue cells on microscopy: stippled vaginal epithelial cells
vaginal pH > 4.5
positive whiff test (addition of potassium hydroxide results in fishy odour)

Management
oral metronidazole for 5-7 days
70-80% initial cure rate
relapse rate > 50% within 3 months
the BNF suggests topical metronidazole or topical clindamycin as alternatives

Bacterial vaginosis in pregnancy
results in an increased risk of preterm labour, low birth weight and chorioamnionitis, late miscarriage
it was previously taught that oral metronidazole should be avoided in the first trimester and topical clindamycin used instead. Recent guidelines however recommend that oral metronidazole is used throughout pregnancy. The BNF still advises against the use of high dose metronidazole regimes

93
Q

You are considering inserting an intrauterine device for emergency contraception. Which one of the following best describes the time limitations from the episode of unprotected sexual intercouse (UPSI)?

Must be inserted within 7 days of UPSI
Must be inserted within 5 days of UPSI OR up to 5 days after the likely ovulation date
Must be inserted within 7 days of UPSI OR up to 10 days after the likely ovulation date
Must be inserted within 7 days of UPSI OR up to 7 days after the likely ovulation date
Must be inserted within 5 days of UPSI
A

Must be inserted within 5 days of UPSI OR up to 5 days after the likely ovulation date

Emergency contraception

There are two methods currently available in the UK:

Emergency hormonal contraception

There are now two methods of emergency hormonal contraception (‘emergency pill’, ‘morning-after pill’); levonorgestrel and ulipristal, a progesterone receptor modulator.

Levonorgestrel
should be taken as soon as possible - efficacy decreases with time
must be taken within 72 hrs of unprotected sexual intercourse (UPSI)*
single dose of levonorgestrel 1.5mg (a progesterone)
mode of action not fully understood - acts both to stop ovulation and inhibit implantation
84% effective is used within 72 hours of UPSI
levonorgestrel is safe and well tolerated. Disturbance of the current menstrual cycle is seen in a significant minority of women. Vomiting occurs in around 1%
if vomiting occurs within 2 hours then the dose should be repeated
can be used more than once in a menstrual cycle if clinically indicated

Ulipristal
a progesterone receptor modulator currently marketed as EllaOne. The primary mode of action is thought to be inhibition of ovulation
30mg oral dose taken as soon as possible, no later than 120 hours after intercourse
concomitant use with levonorgestrel is not recommended
may reduce the effectiveness of combined oral contraceptive pills and progesterone only pills
caution should be exercised in patients with severe asthma
repeated dosing within the same menstrual cycle is not recommended
breastfeeding should be delayed for one week after taking ulipristal. There are no such restrictions on the use of levonorgestrel

Intrauterine device (IUD)
must be inserted within 5 days of UPSI, or
if a women presents after more than 5 days then an IUD may be fitted up to 5 days after the likely ovulation date
may inhibit fertilisation or implantation
prophylactic antibiotics may be given if the patient is considered to be at high-risk of sexually transmitted infection
is 99% effective regardless of where it is used in the cycle
may be left in-situ to provide long-term contraception. If the client wishes for the IUD to be removed it should be at least kept in until the next period

*may be offered after this period as long as the client is aware of reduced effectiveness and unlicensed indication

94
Q

Catherine is 27 year old lady who presents requesting a termination of pregnancy. She is 8 weeks pregnant and this is her first pregnancy. She is concerned with the possible consequences of a single surgical termination of pregnancy, in particular future fertility. Which of the following is correct guidance?

No evidence of impact on future fertility
Increased risk of placenta praevia
Increased risk of ectopic pregnancy
Increased risk of still born
Increased risk of miscarriage
A

Catherine can be informed that there is no evidence of an impact on future fertility and no association with placenta praevia, ectopic pregnancy, still born or miscarriage.

The following guidance has been taken from the Royal College of Obstetrics and Gynaecology entitled ‘The Care of Women Requesting Induced Abortion, Evidence-based Clinical Guideline Number 7’ :

Women should be informed of the uncommon complications that may occur and of their
possible clinical consequences. These may include:

Severe bleeding requiring transfusion: the risk is lower for early abortions, occurring in less
than 1 in 1000, rising to around 4 in 1000 at gestations beyond 20 weeks.
Uterine perforation (surgical abortion only): the risk is in the order of 14 in 1000 and is
lower for early abortions and those performed by experienced clinicians.
Cervical trauma (surgical abortion only): the risk of damage to the external os is no greater
than 1 in 100 and is lower for early abortions and those performed by experienced clinicians.
Women must be informed that, should one of these complications occur, further treatment in
the form of blood transfusion, laparoscopy or laparotomy may be required.

Women should be informed that infection of varying degrees of severity may occur after
medical or surgical abortion and is usually caused by pre-existing infection. Prophylactic
antibiotic use and bacterial screening for lower genital tract infection reduces this risk

Reference:

The Care of Women Requesting Induced Abortion, Evidence-based Clinical Guideline Number 7 https://www.rcog.org.uk/globalassets/documents/guidelines/abortion-guidelineweb1.pdf

Termination of pregnancy

The current law surround abortion is based on the 1967 Abortion Act. In 1990 the act was amended, reducing the upper limit from 28 weeks gestation to 24 weeks*

Key points
two registered medical practitioners must sign a legal document (in an emergency only one is needed)
only a registered medical practitioner can perform an abortion, which must be in a NHS hospital or licensed premise

The method used to terminate pregnancy depend upon gestation
less than 9 weeks: mifepristone (an anti-progestogen, often referred to as RU486) followed 48 hours later by prostaglandins to stimulate uterine contractions
less than 13 weeks: surgical dilation and suction of uterine contents
more than 15 weeks: surgical dilation and evacuation of uterine contents or late medical abortion (induces ‘mini-labour’)

*these limits do not apply in cases where it is necessary to save the life of the woman, there is evidence of extreme fetal abnormality, or there is risk of serious physical or mental injury to the woman.

95
Q

A 34-year-old woman is reviewed. She complains of a long history of deep pain during intercourse and painful periods. There is a previous history of lower abdominal pain and in the past she has been diagnosed with irritable bowel syndrome. She is upset because she would like to start a family but the pain is putting her off sex. What is the most suitable management?

Trial of combined oral contraceptive pill
Discuss benefits of intrauterine system
Refer to gynaecology
Refer for psychosexual counselling
Arrange pelvic ultrasound
A

The combination of deep dyspareunia and lower abdominal pain make a diagnosis of endometriosis likely. Initial treatment options such as the combined pill are not an option in a woman trying to conceive. For a definitive diagnosis the patient should ideally have a laparoscopy. A pelvic ultrasound is not the investigation of choice in endometriosis and may be normal with mild-moderate disease

Endometriosis

Endometriosis is a common condition characterised by the growth of ectopic endometrial tissue outside of the uterine cavity. Up to 10-15% of women have a degree of endometriosis

Clinical features
chronic pelvic pain
dysmenorrhoea - pain often starts days before bleeding
deep dyspareunia
subfertility

Less common features
urinary symptoms e.g. dysuria, urgency
dyschezia (painful bowel movements)

Investigation
laparoscopy is the gold-standard investigation
there is little role for investigation in primary care (e.g. ultrasound)- if the symptoms are significant the patient should be referred for a definitive diagnosis

Management depends on clinical features - there is poor correlation between laparoscopic findings and severity of symptoms
NSAIDs and other analgesia for symptomatic relief
combined oral contraceptive pill
progestogens e.g. medroxyprogesterone acetate
gonadotrophin-releasing hormone (GnRH) analogues - said to induce a ‘pseudomenopause’ due to the low oestrogen levels
intrauterine system (Mirena)
drug therapy unfortunately does not seem to have a significant impact on fertility rates

Surgery
some treatments such as laparoscopic excision and laser treatment of endometriotic ovarian cysts may improve fertility

96
Q

An antenatal ultrasound scan demonstrates a hyperechogenic bowel. Which one of the following is this finding most associated with?

Omphalocele
Polyhydramnios
Multiple pregnancy
Gastroschisis
Cystic fibrosis
A

Cystic fibrosis

Ultrasound in pregnancy

A nuchal scan is performed at 11-13 weeks. Causes of an increased nuchal translucency include:
Down’s syndrome
congenital heart defects
abdominal wall defects

Causes of hyperechogenic bowel:
cystic fibrosis
Down’s syndrome
cytomegalovirus infection

97
Q

A 29-year-old woman loses around 1,000 ml of blood shortly following a vaginal delivery. Which one of the following is not a risk factor for primary post-partum haemorrhage?

Macrosomia
Pre-eclampsia
Afro-Caribbean ethnicity
Prolonged labour
Polyhydramnios
A

Afro-Caribbean ethnicity

Post-partum haemorrhage

Post-partum haemorrhage (PPH) is defined as blood loss of > 500mls and may be primary or secondary

Primary PPH
occurs within 24 hours
affects around 5-7% of deliveries
most common cause of PPH is uterine atony (90% of cases). Other causes include genital trauma and clotting factors

Risk factors for primary PPH include*:
previous PPH
prolonged labour
pre-eclampsia
increased maternal age
polyhydramnios
emergency Caesarean section
placenta praevia
macrosomia
ritodrine (a beta-2 adrenergic receptor agonist used for tocolysis)

Management
ABC
IV syntocinon (oxytocin) 10 units or IV ergometrine 500 mcg
IM carboprost
other options include: B-Lynch suture, ligation of the uterine arteries or internal iliac arteries
if severe, uncontrolled haemorrhage then a hysterectomy is sometimes performed as a life-saving procedure

Secondary PPH
occurs between 24 hours - 12 weeks**
due to retained placental tissue or endometritis

*the effect of parity on the risk of PPH is complicated. It was previously though multiparity was a risk factor but more modern studies suggest nulliparity is actually a risk factor

**previously the definition of secondary PPH was 24 hours - 6 weeks. Please see the RCOG guidelines for more details

98
Q

A couple who are struggling to conceive present to surgery for review. The results of a semen sample are discussed. Which one of the following values, if any, is abnormal?

Sperm concentration 27 million / ml
Morphology 39% normal forms
59% progressive motility
Volume 3 ml
None of the above
A

None of the above

The values listed are within the normal reference ranges

Semen analysis

Semen analysis should be performed after a minimum of 3 days and a maximum of 5 days abstinence. The sample needs to be delivered to the lab within 1 hour

Normal semen results*
volume > 1.5 ml
pH > 7.2
sperm concentration > 15 million / ml
morphology > 4% normal forms
motility > 32% progressive motility
vitality > 58% liver spermatozoa

*many different reference ranges exist. These are based on the NICE 2013 values

99
Q

A 19-year-old female is reviewed following a medical termination of pregnancy (estimated gestation 8 weeks). She is interested in having a Nexplanon inserted. At what point can a Nexplanon be inserted in these circumstances?

Immediately
After 7 days
After 4 weeks
After 6 weeks
After 12 weeks
A

Immediately

Implantable contraceptives

Implanon was a non-biodegradable subdermal contraceptive implant which has been replaced by Nexplanon. From a pharmacological perspective Nexplanon is the same as Implanon. The two main differences are:
the applicator has been redesigned to try and prevent ‘deep’ insertions (i.e. subcutaneous/intramuscular)
it is radiopaque and therefore easier to locate if impalpable

Both versions slowly releases the progestogen hormone etonogestrel. They are typically inserted in the proximal non-dominant arm, just overlying the tricep. The main mechanism of action is preventing ovulation. They also work by thickening the cervical mucus.

Key points
highly effective: failure rate 0.07/100 women/year
long-acting: lasts 3 years
doesn’t contain oestrogen so can be used if past history of thromboembolism, migraine etc
can be inserted immediately following a termination of pregnancy

Disadvantages include
the need for a trained professional to insert and remove device
additional contraceptive methods are needed for the first 7 days if not inserted on day 1 to 5 of a woman’s menstrual cycle

Adverse effects
irregular/heavy bleeding is the main problem
‘progestogen effects’: headache, nausea, breast pain

Interactions
enzyme-inducing drugs such as certain antiepileptic and rimampicin may reduce the efficacy of Nexplanon
the FSRH advises that women should be advised to switch to a method unaffected by enzyme-inducing drugs or to use additional contraception until 28 days after stopping the treatment

Contraindications
UKMEC 3*: ischaemic heart disease/stroke (for continuation, if initiation then UKMEC 2), unexplained, suspicious vaginal bleeding, past breast cancer, severe liver cirrhosis, liver cancer, positive antiphospholipid antibodies**
UKMEC 4**: current breast cancer

proven risks generally outweigh the advantages
**there is some contradiction in the guidance issued by the FSRH but their most recent document (revised 2010) lists positive antiphospholipid antibodies as UKMEC 3
**
a condition which represents an unacceptable risk if the contraceptive method is used

100
Q

Which one of the following prescriptions is contraindicated in pregnancy?

Methyldopa for hypertension
Topical clindamycin for bacterial vaginosis
Doxycycline for malarial prophylaxis
Metoclopramide for vomiting
Prednisolone for an asthma exacerbation
A

All tetracyclines should be avoided in pregnancy.

It should be noted that the above prescriptions are not necessarily the recommended first-line treatments

Prescribing in pregnant patients

Very few drugs are known to be completely safe in pregnancy. The list below largely comprises of those known to be harmful. Some countries have developed a grading system - see the link.

Antibiotics
tetracyclines
aminoglycosides
sulphonamides and trimethoprim
quinolones: the BNF advises to avoid due to arthropathy in some animal studies

Other drugs
ACE inhibitors, angiotensin II receptor antagonists
statins
warfarin
sulfonylureas
retinoids (including topical)
cytotoxic agents

The majority of antiepileptics including valproate, carbamazepine and phenytoin are known to be potentially harmful. The decision to stop such treatments however is difficult as uncontrolled epilepsy is also a risk

101
Q

Which one of the following contraceptives may decrease bone mineral density in women?

Depo Provera (injectable contraceptive)
Progestogen only pill
Mirena (intrauterine system)
Implanon (implantable contraceptive)
Combined oral contraceptive pill
A

Depo Provera (injectable contraceptive)

Injectable contraceptives

Depo Provera is the main injectable contraceptive used in the UK*. It contains medroxyprogesterone acetate 150mg. It is given via in intramuscular injection every 12 weeks. It can however be given up to 14 weeks after the last dose without the need for extra precautions**

The main method of action is by inhibiting ovulation. Secondary effects include cervical mucus thickening and endometrial thinning.

Disadvantages include the fact that the injection cannot be reversed once given. There is also a potential delayed return to fertility (maybe up to 12 months)

Adverse effects
irregular bleeding
weight gain
may potentially increased risk of osteoporosis: should only be used in adolescents if no other method of contraception is suitable
not quickly reversible and fertility may return after a varying time

*Noristerat, the other injectable contraceptive licensed in the UK, is rarely used in clinical practice. It is given every 8 weeks

**the BNF gives different advice, stating a pregnancy test should be done if the interval is greater than 12 weeks and 5 days - this is however not commonly adhered to in the family planning community

102
Q

Which one of the following statements regarding epilepsy in pregnant women is correct?

All pregnant women on antiepileptic medication should take 400mcg a day of folic acid
Antiepileptic drug levels should be monitored throughout pregnancy
The dose of lamotrigine usually needs to be decreased during pregnancy
Pregnant women taking phenytoin should be given vitamin K in the last month of pregnancy
Sodium valproate is most strongly associated with cleft palate
A

Pregnant women taking phenytoin should be given vitamin K in the last month of pregnancy

Epilepsy: pregnancy and breast feeding

The risks of uncontrolled epilepsy during pregnancy generally outweigh the risks of medication to the fetus. All women thinking about becoming pregnant should be advised to take folic acid 5mg per day well before pregnancy to minimise the risk of neural tube defects. Around 1-2% of newborns born to non-epileptic mothers have congenital defects. This rises to 3-4% if the mother takes antiepileptic medication.

Other points
aim for monotherapy
there is no indication to monitor antiepileptic drug levels
sodium valproate: associated with neural tube defects
carbamazepine: often considered the least teratogenic of the older antiepileptics
phenytoin: associated with cleft palate
lamotrigine: studies to date suggest the rate of congenital malformations may be low. The dose of lamotrigine may need to be increased in pregnancy

Breast feeding is generally considered safe for mothers taking antiepileptics with the possible exception of the barbiturates

It is advised that pregnant women taking phenytoin are given vitamin K in the last month of pregnancy to prevent clotting disorders in the newborn

Sodium valproate

The November 2013 issue of the Drug Safety Update also carried a warning about new evidence showing a significant risk of neurodevelopmental delay in children following maternal use of sodium valproate.

The update concludes that sodium valproate should not be used during pregnancy and in women of childbearing age unless clearly necessary. Women of childbearing age should not start treatment without specialist neurological or psychiatric advice.

103
Q

A 23-year-old who is 10 weeks pregnant is reviewed by the midwife at the booking visit. This is her first pregnancy and she is well apart from some sickness which is worse in the morning and a generalised pruritus. Bloods tests including the full blood count, hepatitis B, C and HIV serology are normal. A slight yellow tinge of her sclera is noticed and liver function tests are ordered:

Bilirubin 31 µmol/L
ALP 160 U/L
ALT 25 U/L
Albumin 34 g/L

What is the most likely diagnosis?

Gilbert's syndrome
Gallstones
Acute fatty liver
Intrahepatic cholestasis of pregnancy
Primary biliary cirrhosis
A

Morning sickness and pruritus are common in pregnant women. Intrahepatic cholestasis of pregnancy would not occur in the first trimester. An ALP of 160 U/l is normal in a pregnant woman leaving the only abnormal result being the raised bilirubin (which usually falls in pregnancy). The most likely diagnosis is therefore Gilbert’s syndrome.

Gilbert’s syndrome

Gilbert’s syndrome is an autosomal recessive* condition of defective bilirubin conjugation due to a deficiency of UDP glucuronyl transferase. The prevalence is approximately 1-2% in the general population

Features
unconjugated hyperbilinaemia (i.e. not in urine)
jaundice may only be seen during an intercurrent illness

Investigation and management
investigation: rise in bilirubin following prolonged fasting or IV nicotinic acid
no treatment required

*the exact mode of inheritance is still a matter of debate

104
Q

A 30-year-old woman comes for review. She reports always having had heavy periods but over the past six months they have become worse. There is no history of dysmenorrhoea, intermenstrual or postcoital bleeding. She has had two children and says she does not want anymore. Gynaecology examination is normal and her cervical smear is up-to-date. What is the treatment of choice?

Hysterectomy
Combined oral contraceptive pill
Endometrial ablation
Intrauterine system (Mirena)
Tranexamic acid
A

Menorrhagia - intrauterine system (Mirena) is first-line

Menorrhagia: management

Menorrhagia was previously defined as total blood loss > 80 ml per menses, but it is obviously difficult to quantify. The management has therefore shifted towards what the woman considers to be excessive. Prior to the 1990’s many women underwent a hysterectomy to treat heavy periods but since that time the approach has altered radically. The management of menorrhagia now depends on whether a women needs contraception.

Investigations
a full blood count should be performed in all women
further investigations are based upon the history and examination findings

Does not require contraception
either mefenamic acid 500 mg tds (particularly if there is dysmenorrhoea as well) or tranexamic acid 1 g tds. Both are started on the first day of the period
if no improvement then try other drug whilst awaiting referral

Requires contraception, options include
intrauterine system (Mirena) should be considered first-line
combined oral contraceptive pill
long-acting progestogens

Norethisterone 5 mg tds can be used as a short-term option to rapidly stop heavy menstrual bleeding.

105
Q

Which one of the following statements regarding the management of pregnant, non-sensitised Rhesus negative women is correct?

External cephalic version requires anti-D immunoglobulin prophylaxis
Around 30-35% of mothers are Rhesus negative
Fetal hydronephrosis may result if maternal anti-D antibodies cross the placenta
All Rhesus negative women should receive anti-D at 20 and 24 weeks
Anti-D is not required following a medical termination of pregnancy
A

External cephalic version requires anti-D immunoglobulin prophylaxis

Rhesus negative pregnancy

A basic understanding of the pathophysiology is essential to understand the management of Rhesus negative pregnancies
along with the ABO system the Rhesus system is the most important antigen found on red blood cells. The D antigen is the most important antigen of the rhesus system
around 15% of mothers are rhesus negative (Rh -ve)
if a Rh -ve mother delivers a Rh +ve child a leak of fetal red blood cells may occur
this causes anti-D IgG antibodies to form in mother
in later pregnancies these can cross placenta and cause haemolysis in fetus
this can also occur in the first pregnancy due to leaks

Prevention
test for D antibodies in all Rh -ve mothers at booking
NICE (2008) advise giving anti-D to non-sensitised Rh -ve mothers at 28 and 34 weeks
the evidence base suggests that there is little difference in the efficacy of single-dose (at 28 weeks) and double-dose regimes (at 28 & 34 weeks). For this reason the RCOG in 2011 advised that either regime could be used ‘depending on local factors’
anti-D is prophylaxis - once sensitization has occurred it is irreversible
if event is in 2nd/3rd trimester give large dose of anti-D and perform Kleihauer test - determines proportion of fetal RBCs present

Anti-D immunoglobulin should be given as soon as possible (but always within 72 hours) in the following situations:
delivery of a Rh +ve infant, whether live or stillborn
any termination of pregnancy
miscarriage if gestation is > 12 weeks
ectopic pregnancy
external cephalic version
antepartum haemorrhage
amniocentesis, chorionic villus sampling, fetal blood sampling

Tests
all babies born to Rh -ve mother should have cord blood taken at delivery for FBC, blood group & direct Coombs test
Coombs test: direct antiglobulin, will demonstrate antibodies on RBCs of baby
Kleihauer test: add acid to maternal blood, fetal cells are resistant

Affected fetus
oedematous (hydrops fetalis, as liver devoted to RBC production albumin falls)
jaundice, anaemia, hepatosplenomegaly
heart failure
kernicterus
treatment: transfusions, UV phototherapy

106
Q

A 24-year-old woman who is 34 weeks pregnant presents with pleuritic chest pain and shortness of breath. She has noticed some pain in her left calf for the past 3 days and on examination she has clinical signs consistent with a left calf deep vein thrombosis. What is the most appropriate investigation?

D-dimer
Compression duplex Doppler
Computed tomographic pulmonary angiography
Venogram
Ventilation-perfusion scan
A

Confirming a DVT is the first step as this may provide indirect evidence of a pulmonary embolism. As both conditions require anticoagulation this may negate the need for further radiation exposure.

Pregnancy: DVT/PE investigation

Guidelines were updated in 2010 by the Royal College of Obstetricians. Key points include:
chest x-ray should be performed in all patients
- compression duplex Doppler should be performed if the chest x-ray is normal - this may provide indirect evidence of a pulmonary embolism and negate the need for further radiation exposure
the decision to perform a V/Q or CTPA should be taken at a local level after discussion with the patient and radiologist

Comparing CTPA to V/Q scanning in pregnancy

CTPA
CTPA slightly increases the lifetime risk of maternal breast cancer (increased by up to 13.6%, background risk of 1/200 for study population). Pregnancy makes breast tissue particularly sensitive to the effects of radiation

V/Q scanning

V/Q scanning carries a slightly increased risk of childhood cancer compared with CTPA (1/280,000 versus less than 1/1,000,000)

107
Q

Which one of the following statements regarding the link between intrauterine devices (IUDs) and pelvic inflammatory disease (PID) is correct?

Decreased risk in first 20 days then returns to normal
There is no link between IUDs and PID
Overall decreased risk throughout lifetime of IUD
Overall increased risk throughout lifetime of IUD
Increased risk in first 20 days then returns to normal
A

Increased risk in first 20 days then returns to normal

Intrauterine contraceptive devices

Intrauterine contraceptive devices comprise both conventional copper intrauterine devices (IUDs) and levonorgestrel-releasing intrauterine systems (IUS, Mirena). The IUS is also used in the management of menorrhagia

Effectiveness
both the IUD and IUS are more than 99% effective

Mode of action
IUD: primary mode of action is prevention of fertilisation by causing decreased sperm motility and survival (possibly an effect of copper ions)
IUS: levonorgestrel prevents endometrial proliferation and causes cervical mucous thickening

Counselling
IUD is effective immediately following insertion
IUS can be relied upon after 7 days

Potential problems
IUDs make periods heavier, longer and more painful
the IUS is associated with initial frequent uterine bleeding and spotting. Later women typically have intermittent light menses with less dysmenorrhoea and some women become amenorrhoeic
uterine perforation: up to 2 per 1000 insertions
the proportion of pregnancies that are ectopic is increased but the absolute number of ectopic pregnancies is reduced, compared to a woman not using contraception
infection: there is a small increased risk of pelvic inflammatory disease in the first 20 days after insertion but after this period the risk returns to that of a standard population
expulsion: risk is around 1 in 20, and is most likely to occur in the first 3 months

108
Q

A 36-year-old woman who is undergoing her first IVF cycle presents with abdominal pain. Which one of the following would be least consistent with a diagnosis of ovarian hyperstimulation syndrome?

Thromboembolism
Jaundice
Clinical evidence of ascites
Oliguria
Vomiting
A

Jaundice

Ovarian hyperstimulation syndrome

Ovarian hyperstimulation syndrome (OHSS) is a complication seen in some forms of infertility treatment. It is postulated that the presence of multiple luteinized cysts within the ovaries results in high levels of not only oestrogens and progesterone but also vasoactive substances such as vascular endothelial growth factor (VEGF). This results in increased membrane permeability and loss of fluid from the intravascular compartment

Whilst it is rarely seen with clomifene therapy is more likely to be seen following gonadotropin or hCG treatment. Up to one third of women who are having IVF may experience a mild form of OHSS

109
Q

A 29-year-old woman has just found out she is pregnant for the second time. Her first pregnancy was complicated by gestational diabetes. Following her first pregnancy she was told she was no longer diabetic. What is the most appropriate management?

Start insulin
Start metformin and do oral glucose tolerance test at 12-14 weeks
Do oral glucose tolerance test at booking visit
Do oral glucose tolerance test at 16-18 weeks
Do fasting glucose at booking visit
A

Do oral glucose tolerance test at 16-18 weeks

Pregnancy: diabetes mellitus

Diabetes mellitus may be a pre-existing problem or develop during pregnancy, gestational diabetes. It complicates around 1 in 40 pregnancies

Risk factors for gestational diabetes
BMI of > 30 kg/m^2
previous macrosomic baby weighing 4.5 kg or above.
previous gestational diabetes
first-degree relative with diabetes
family origin with a high prevalence of diabetes (South Asian, black Caribbean and Middle Eastern)

Screening for gestational diabetes
if a women has had gestational diabetes previously an oral glucose tolerance test (OGTT) should be performed at 16-18 weeks and at 28 weeks if the first test is normal
women with any of the other risk factors should be offered an OGTT at 24-28 weeks
currently the same WHO diagnostic criteria are used as for non-pregnant patients. There is however increasing evidence that a lower threshold should be used as treating borderline patients improves both maternal and neonatal outcomes

NICE issued guidelines on the management of diabetes mellitus in pregnancy which were updated in 2008

Management of pre-existing diabetes
weight loss for women with BMI of > 27 kg/m^2
stop oral hypoglycaemic agents, apart from metformin, and commence insulin
folic acid 5 mg/day from pre-conception to 12 weeks gestation
detailed anomaly scan at 18-20 weeks including four-chamber view of the heart and outflow tracts
tight glycaemic control reduces complication rates
treat retinopathy as can worsen during pregnancy

Management of gestational diabetes
responds to changes in diet and exercise in around 80% of women
oral hypoglycaemic agents (metformin or glibenclamide) or insulin injections are needed if blood glucose control is poor or this is any evidence of complications (e.g. macrosomia)
there is increasing evidence that oral hypoglycaemic agents are both safe and give similar outcomes to insulin
hypoglycaemic medication should be stopped following delivery
a fasting glucose should be checked at the 6 week postnatal check

110
Q

An 18-year-old female presents with post-coital bleeding. As part of your management you consider testing the patient for Chlamydia. What percentage of young women in the UK have Chlamydia?

Approximately 0.5%
Approximately 2%
Approximately 5%
Approximately 10%
Approximately 25%
A

Chlamydia is common, affecting around 1 in 10 young women

Chlamydia

Chlamydia is the most prevalent sexually transmitted infection in the UK and is caused by Chlamydia trachomatis, an obligate intracellular pathogen. Approximately 1 in 10 young women in the UK have Chlamydia. The incubation period is around 7-21 days, although it should be remembered a large percentage of cases are asymptomatic

Features
asymptomatic in around 70% of women and 50% of men
women: cervicitis (discharge, bleeding), dysuria
men: urethral discharge, dysuria

Potential complications
epididymitis
pelvic inflammatory disease
endometritis
increased incidence of ectopic pregnancies
infertility
reactive arthritis
perihepatitis (Fitz-Hugh-Curtis syndrome)

Investigation
traditional cell culture is no longer widely used
nuclear acid amplification tests (NAATs) are now rapidly emerging as the investigation of choice
urine (first void urine sample), vulvovaginal swab or cervical swab may be tested using the NAAT technique

Screening
in England the National Chlamydia Screening Programme is open to all men and women aged 15-24 years
the 2009 SIGN guidelines support this approach, suggesting screening all sexually active patients aged 15-24 years
relies heavily on opportunistic testing

Management
doxycycline (7 day course) or azithromycin (single dose). The 2009 SIGN guidelines suggest azithromycin should be used first-line due to potentially poor compliance with a 7 day course of doxycycline
if pregnant then erythromycin or amoxicillin may be used. The SIGN guidelines suggest considering azithromycin ‘following discussion of the balance of benefits and risks with the patient’
patients diagnosed with Chlamydia should be offered a choice of provider for initial partner notification - either trained practice nurses with support from GUM, or referral to GUM
for men with symptomatic infection all partners from the four weeks prior to the onset of symptoms should be contacted
for women and asymptomatic men all partners from the last six months or the most recent sexual partner should be contacted
contacts of confirmed Chlamydia cases should be offered treatment prior to the results of their investigations being known (treat then test)

111
Q

A 16-year-old female with a history of acne requests to start a combined oral contraceptive pill (COC). She has been taking oxytetracycline for the past 2 months. What is the most appropriate advice?

A double-dose COC should be used
Condoms should be used for the first 14 days of COC use
She can start using a COC with usual advice
A COC is inappropriate and an alternative method of contraception should be considered
Condoms should be used for the first 21 days of COC use
A

The usual rules regarding starting the COC should still be adhered to, i.e. use condoms for 7 days if not started on the first day of next period

Combined oral contraceptive pill: special situations

Concurrent antibiotic use
for many years doctors in the UK have advised that the concurrent use of antibiotics may interfere with the enterohepatic circulation of oestrogen and thus make the combined oral contraceptive pill ineffective - ‘extra- precautions’ were advised for the duration of antibiotic treatment and for 7 days afterwards
no such precautions are taken in the US or the majority of mainland Europe
in 2011 the Faculty of Sexual & Reproductive Healthcare produced new guidelines abandoning this approach. The latest edition of the BNF has been updated in line with this guidance
precautions should still be taken with enzyme inducing antibiotics such as rifampicin

Switching combined oral contraceptive pills
the BNF and Faculty of Sexual & Reproductive Healthcare (FSRH) appear to give contradictory advice. The Clinical Effectiveness Unit of the FSRH have stated in the Combined Oral Contraception guidelines that the pill free interval does not need to be omitted (please see link). The BNF however advises missing the pill free interval if the progesterone changes. Given the uncertainty it is best to follow the BNF

112
Q

Theme: Urinary incontinence

A. Bladder diary for 3 days
B. Urodynamic studies
C. Bladder drill training for 6 weeks
D. Pelvic floor exercises for 3 months
E. Oxybutynin
F. IV urography
G. Dyes studies including phenazopyridine
H. None of the above

Choose the best management option for each clinical scenario. Each option may be used once, more than once or not at all.

A 75-year-old lady reports urinary incontinence when coughing and sneezing. She has had 2 children with no complications. She has no significant past medical history and is on no medications. What is the most appropriate initial management?

A

Pelvic floor exercises for 3 months

A diagnosis of stress incontinence is obvious from the history, therefore there is no need for a bladder diary or urodynamic studies.
Pelvic floor exercises would be the first line management.

113
Q

Theme: Urinary incontinence

A. Bladder diary for 3 days
B. Urodynamic studies
C. Bladder drill training for 6 weeks
D. Pelvic floor exercises for 3 months
E. Oxybutynin
F. IV urography
G. Dyes studies including phenazopyridine
H. None of the above

Choose the best management option for each clinical scenario. Each option may be used once, more than once or not at all.

A 26-year-old pregnant woman having her first child and has never had problems with incontinence.

A

Pelvic floor exercises for 3 months

Pregnant women should receive instructions as to how to perform pelvic floor exercises during pregnancy as this may help to decrease subsequent risk of stress urinary incontinence.

114
Q

Theme: Urinary incontinence

A. Bladder diary for 3 days
B. Urodynamic studies
C. Bladder drill training for 6 weeks
D. Pelvic floor exercises for 3 months
E. Oxybutynin
F. IV urography
G. Dyes studies including phenazopyridine
H. None of the above

Choose the best management option for each clinical scenario. Each option may be used once, more than once or not at all.

A 67-year-old lady reports urinary incontinence. She describes the sensation of needing to pass urine immediately. She has had 2 children and is on no medications. What is the most appropriate initial management?

A

Bladder diary for 3 days

The patient appears to be describing urge incontinence. A bladder diary is needed to establish the baseline frequency of micturition and amounts of urine passed. Then bladder training can be initiated to increase the volume of urine passed at reduced frequencies.

115
Q

A 28-year-old woman with rheumatoid arthritis asks for advice about conception. Which one of the following statements is true?

Methotrexate may be continued during pregnancy as long as the woman takes folic acid 5mg daily
NSAIDs should be avoided in the first and second trimester
Woman with rheumatoid should be encouraged to conceive as soon as possible (ideally within 1 year) after diagnosis to minimise the risk of complications
TNF-α blockers are absolutely contraindicated in pregnancy
Hydroxychloroquine is considered safe during pregnancy
A

Hydroxychloroquine is considered safe during pregnancy

Rheumatoid arthritis: pregnancy

Rheumatoid arthritis (RA) typically develops in women of a reproductive age. Issues surrounding conception are therefore commonly encountered. There are no current published guidelines regarding how patients considering conception should be managed although expert reviews are largely in agreement.

Key points
patients with early or poorly controlled RA should be advised to defer conception until their disease is more stable
RA symptoms tend to improve in pregnancy but only resolve in a small minority. Patients tend to have a flare following delivery
methotrexate is not safe in pregnancy and needs to be stopped at least 3 months before conception
leflunomide is not safe in pregnancy
sulfasalazine and hydroxychloroquine are considered safe in pregnancy
interestingly studies looking at pregnancy outcomes in patients treated with TNF-α blockers do not show any significant increase in adverse outcomes. It should be noted however that many of the patients included in the study stopped taking TNF-α blockers when they found out they were pregnant
low-dose corticosteroids may be used in pregnancy to control symptoms
NSAIDs may be used until 32 weeks but after this time should be withdrawn due to the risk of early close of the ductus arteriosus
patients should be referred to an obstetric anaesthetist due to the risk of atlanto-axial subluxation

116
Q

Which one of the following is an absolute contraindication to combined oral contraceptive pill use?

Known thrombogenic mutation
Family history of thromboembolic disease in first degree relatives
A

Known thrombogenic mutation

Combined oral contraceptive pill: contraindications

The decision of whether to start a women on the combined oral contraceptive pill is now guided by the UK Medical Eligibility Criteria (UKMEC). This scale categorises the potential cautions and contraindications according to a four point scale, as detailed below:
UKMEC 1: a condition for which there is no restriction for the use of the contraceptive method
UKMEC 2: advantages generally outweigh the disadvantages
UKMEC 3: disadvantages generally outweigh the advantages
UKMEC 4: represents an unacceptable health risk

Examples of UKMEC 3 conditions include
more than 35 years old and smoking less than 15 cigarettes/day
BMI > 35 kg/m^2*
migraine without aura and more than 35 years old
family history of thromboembolic disease in first degree relatives 20 years ago is classified as UKMEC 3 or 4 depending on severity

*The UKMEC 4 rating for a BMI > 40 kg/m^2 was removed in 2009.

117
Q

Which one of the following statements regarding amniocentesis is incorrect?

Is performed under ultrasound guidance
Is usually performed at 16 weeks
Can help detect neural tube defects
Karyotype may be wrong in 1 in 1,000 cases
Risk of fetal loss is 2-3%
A

Amniocentesis: risk of fetal loss is 0.5-1%

Amniocentesis

Amniocentesis is a procedure used in prenatal diagnosis. It may be offered after screening tests have indicated a high risk of fetal abnormality or in women considered to be at high risk, for example if > 35 years old.

Around 20 ml of fluid is removed by transabdominal needle under ultrasound guidance. Fetal cells present in the amniotic fluid are then studied to aid the diagnosis of a number of conditions.

Amniocentesis is usually performed at 16 weeks and the risk of fetal loss is 0.5-1%. The karyotype results typically take 3 weeks. It is known the karyotype may be wrong in 1/1000 cases due to maternal cells being present

Conditions which may be diagnosed
neural tube defects (raised AFP levels in the amniotic fluid)
chromosomal disorders
inborn errors of metabolism

118
Q

A 33-year-old female presents to her GP as she missed her Noriday pill (progestogen only) this morning and is unsure what to do. She normally takes the pill at around 0900 and it is now 1230. What advice should be given?

Take missed pill as soon as possible and advise condom use until pill taking re-established for 48 hours
Take missed pill as soon as possible and omit pill break at end of pack
Perform a pregnancy test
Take missed pill as soon as possible and no further action needed
Emergency contraception should be offered
A

Progestogen only pill: missed pill

The missed pill rules for the progestogen only pill (POP) are simpler than those used for the combined oral contraceptive pill, but it is important not to confuse the two.

‘Traditional’ POPs (Micronor, Noriday, Nogeston, Femulen)

If less than 3 hours late
no action required, continue as normal

If more than 3 hours late (i.e. more than 27 hours since the last pill was taken)
action needed - see below

Cerazette (desogestrel)

If less than 12 hours late
no action required, continue as normal

If more than 12 hours late (i.e. more than 36 hours since the last pill was taken)
action needed - see below

Action required, if needed:
take the missed pill as soon as possible. If more than one pill has been missed just take one pill. Take the next pill at the usual time, which may mean taking two pills in one day
continue with rest of pack
extra precautions (e.g. condoms) should be used until pill taking has been re-established for 48 hours

119
Q

A 29-year-old woman who is 28 weeks pregnant is reviewed. She has developed pre-eclampsia with her current blood pressure being 156/104 mmHg and the urine dipstick reported as follows:

Protein +
Leucocytes negative
Blood negative

There is no oedema and the patient is otherwise asymptomatic. Of the following drugs, which one is least suitable to use?

Labetalol
Nifedipine
Losartan
Methyldopa
Hydralazine
A

ACE inhibitors and angiotensin-2 receptor blockers should be avoided as they are teratogenic. Most clinicians would either use methyldopa or labetalol first-line in this situation

Pre-eclampsia

Pre-eclampsia is a condition seen after 20 weeks gestation characterised by pregnancy-induced hypertension in association with proteinuria (> 0.3g / 24 hours). Oedema used to be third element of the classic triad but is now often not included in the definition as it is not specific

Pre-eclampsia is important as it predisposes to the following problems
fetal: prematurity, intrauterine growth retardation
eclampsia
haemorrhage: placental abruption, intra-abdominal, intra-cerebral
cardiac failure
multi-organ failure

Risk factors
> 40 years old
nulliparity (or new partner)
multiple pregnancy
body mass index > 30 kg/m^2
diabetes mellitus
pregnancy interval of more than 10 years
family history of pre-eclampsia
previous history of pre-eclampsia
pre-existing vascular disease such as hypertension or renal disease

Features of severe pre-eclampsia
hypertension: typically > 170/110 mmHg and proteinuria as above
proteinuria: dipstick ++/+++
headache
visual disturbance
papilloedema
RUQ/epigastric pain
hyperreflexia
platelet count 160/110 mmHg although many clinicians have a lower threshold
oral labetalol is now first-line following the 2010 NICE guidelines. Nifedipine and hydralazine may also be used
delivery of the baby is the most important and definitive management step. The timing depends on the individual clinical scenario

120
Q

A 54-year-old woman is investigated due to a one month history of painless vaginal bleeding. Her last period was three years ago. Which one of the following is a risk factor for endometrial cancer?

Combined oestrogen/progestogen hormone replacement therapy (continuous preparation)
Working in the dye industry
Multiparity
Combined oral contraceptive use in the past
Polycystic ovarian syndrome
A

Polycystic ovarian syndrome

Endometrial cancer

Endometrial cancer is classically seen in post-menopausal women but around 25% of cases occur before the menopause. It usually carries a good prognosis due to early detection

The risk factors for endometrial cancer are as follows*:
obesity
nulliparity
early menarche
late menopause
unopposed oestrogen. The addition of a progestogen to oestrogen reduces this risk (e.g. In HRT). The BNF states that the additional risk is eliminated if a progestogen is given continuously
diabetes mellitus
tamoxifen
polycystic ovarian syndrome

Features
post-menopausal bleeding is the classic symptom
pre-menopausal women may have a change intermenstrual bleeding
pain and discharge are unusual features

Investigation
first-line investigation is trans-vaginal ultrasound - a normal endometrial thickness (

121
Q

A 27-year-old woman has just found out she is pregnant. There is no past medical history of note and this is her first pregnancy. She asks for advice about vitamin D supplementation. What is the most appropriate advice to give?

Offer vitamin D supplementation
Should be avoided - potential risk to developing fetus
Await booking bloods for confirmation of vitamin D levels
Advise to increase milk and soft-cheese consumption e.g. Brie and Stilton
Advise her she can take supplements if she wishes but this is not part of routine NHS antenatal advice
A

Offer vitamin D supplementation

Soft-cheese should be avoided during pregnancy due to the risk of Listeria

Antenatal care: specific points

NICE issued guidelines on routine care for the healthy pregnant woman in March 2008

Nausea and vomiting
natural remedies - ginger and acupuncture on the ‘p6’ point (by the wrist) are recommended by NICE
antihistamines should be used first-line (BNF suggests promethazine as first-line)

Vitamin D
NICE recommend ‘All women should be informed at the booking appointment about the importance for their own and their baby’s health of maintaining adequate vitamin D stores during pregnancy and whilst breastfeeding’
‘women may choose to take 10 micrograms of vitamin D per day, as found in the Healthy Start multivitamin supplement’. This was confirmed in 2012 when the Chief Medical Officer advised: ‘All pregnant and breastfeeding women should take a daily supplement containing 10micrograms of vitamin D, to ensure the mothers requirements for vitamin D are met and to build adequate fetal stores for early infancy’
particular care should be taken with women at risk (e.g. Asian, obese, poor diet)

Alcohol
NICE recommend women should avoid alcohol during the first trimester
if women choose to drink alcohol during pregnancy they should be advised to drink no more than 1 to 2 UK units once or twice a week

122
Q

A 24 year old female attends for her 6 week post natal check up. She is not breast feeding. She had a normal delivery. She would like to start contraception but is concerned about a delay in returning to fertility as she would like to try for another baby in the next 1-2 years. Which of these would most like cause a delay in returning to normal fertility?

Progesterone only contraceptive implant
Intrauterine system (IUS) e.g. mirena
Progesterone only injectable contraception (depo)
Combined oral contraceptive pill (COCP)
Progesterone only pill (POP)
A

With progesterone only injectable contraception there can be a delay in return to natural fertility of up to 12 months. The other methods are not associated with such a delay.

See FSRH guidelines.

Injectable contraceptives

Depo Provera is the main injectable contraceptive used in the UK*. It contains medroxyprogesterone acetate 150mg. It is given via in intramuscular injection every 12 weeks. It can however be given up to 14 weeks after the last dose without the need for extra precautions**

The main method of action is by inhibiting ovulation. Secondary effects include cervical mucus thickening and endometrial thinning.

Disadvantages include the fact that the injection cannot be reversed once given. There is also a potential delayed return to fertility (maybe up to 12 months)

Adverse effects
irregular bleeding
weight gain
may potentially increased risk of osteoporosis: should only be used in adolescents if no other method of contraception is suitable
not quickly reversible and fertility may return after a varying time

*Noristerat, the other injectable contraceptive licensed in the UK, is rarely used in clinical practice. It is given every 8 weeks

**the BNF gives different advice, stating a pregnancy test should be done if the interval is greater than 12 weeks and 5 days - this is however not commonly adhered to in the family planning community

123
Q

A 35-year-old female presents to her GP as she was late taking her Cerazette (desogestrel) pill this morning. Up to what length of time may the taking of Cerazette be delayed?

3 hours
6 hours
9 hours
12 hours
18 hours
A

Progestogen only pill: types

Second generation
norethisterone
levonorgestrel
ethynodiol diacetate

Third generation
desogestrel (Cerazette)

Cerazette
new third generation type of progestogen only pill (POP) containing desogestrel
inhibits ovulation in the majority of women
users can take the pill up to 12 hours late rather than 3 hours like other POPs

124
Q

A 33-year-old woman is reviewed regarding her asthma control. You notice from her records that she has never had a cervical smear and raise this with her. She responds that she is a lesbian and has never had sex with a man. What is the most appropriate advice to give?

She does not need to have a smear
She may need to have a smear if her partner has previously had heterosexual relationships
She should have cervical screening as per normal
She does not need to have a smear but does need a one-off HPV test
She should be referred to colposcopy clinic for a case-based assessment
A

HPV, the causative agent of cervical cancer, can be transmitted during lesbian sex. Lesbian and bisexual women should therefore have cervical screening as normal. Unfortunately the uptake amongst lesbian women is around 10 times worse than the general female population, sometimes as a consequence of incorrect advice from healthcare professionals.

Cervical cancer screening

The UK has a well established cervical cancer screening program which is estimated to prevent 1,000-4,000 deaths per year. It should be noted that cervical adenocarcinomas, which account for around 15% of cases, are frequently undetected by screening

Who is screened and how often?

A smear test is offered to all women between the ages of 25-64 years
25-49 years: 3-yearly screening
50-64 years: 5-yearly screening

How is performed?

There is currently a move away from traditional Papanicolaou (Pap) smears to liquid-based cytology (LBC). Rather than smearing the sample onto a slide the sample is either rinsed into the preservative fluid or the brush head is simply removed into the sample bottle containing the preservative fluid.

Advantages of LBC includes
reduced rate of inadequate smears
increased sensitivity and specificity

It is said that the best time to take a cervical smear is around mid-cycle. Whilst there is limited evidence to support this it is still the current advice given out by the NHS.

In Scotland women from the ages of 20-60 years are screened every 3 years.

125
Q

A 31-year-old woman returns for review. She presented last month with concerns about infertility. She has a 28 day cycle and a day 21 progesterone level was arranged:

Progesterone 34 nmol/l

Which one of the following is correct?

Sample was taken at the wrong time
Sample should be repeated next cycle
Provides evidence of anovulatory cycles
Provides evidence of adequate ovulation
May indicate polycystic ovarian syndrome
A

Provides evidence of adequate ovulation

Infertility

Infertility affects around 1 in 7 couples. Around 84% of couples who have regular sex will conceive within 1 year, and 92% within 2 years

Causes
male factor 30%
unexplained 20%
ovulation failure 20%
tubal damage 15%
other causes 15%

Basic investigations
semen analysis
serum progesterone 7 days prior to expected next period

126
Q

Which one of the following statements regarding preterm birth is incorrect?

Is defined as delivery of an infant before 37 weeks gestation
Around 45% of twin pregnancies are premature
Occurs in around 15-20% of pregnancies
Diabetes mellitus is a risk factor
May be elective due to congenital abnormalities
A

Preterm birth occurs in around 5-10% of pregnancies

Preterm birth

Preterm birth is defined as delivery of an infant before 37 weeks gestation. It occurs in around 5-10% of pregnancies (6% of singletons, 45% of twins)

Causes
unexplained (30-40%)
multiple pregnancies (20-30%)
congenital abnormalities
antepartum haemorrhage
pre-eclampsia
cervical incompetence
diabetes mellitus
polyhydramnios
uterine abnormalities
infections e.g. Pyelonephritis

127
Q

A 34-year-old woman who has never had a cervical smear presents with post-coital bleeding. Which one of the following is not a recognised risk factor for cervical cancer?

Combined oral contraceptive pill use
Obesity
Human papilloma virus 16
Human immunodeficiency virus
High parity
A

Obesity is a risk factor for endometrial cancer but not cervical cancer

Cervical cancer

The incidence of cervical cancer peaks around the 6th decade. It may be divided into
squamous cell cancer (80%)
adenocarcinoma (20%)

Features
may be detected during routine cervical cancer screening
abnormal vaginal bleeding: postcoital, intermenstrual or postmenopausal bleeding
vaginal discharge

Risk factors
human papilloma virus 16,18 & 33
smoking
human immunodeficiency virus
early first intercourse, many sexual partners
high parity
lower socioeconomic status
combined oral contraceptive pill*

*the strength of this association is sometimes debated but a large study published in the Lancet (2007 Nov 10;370(9599):1609-21) confirmed the link

128
Q

Theme: Urinary incontinence

A. Bladder diary for 3 days
B. Urodynamic studies
C. Bladder drill training for 6 weeks
D. Pelvic floor exercises 3 months
E. Oxybutynin
F. IV urography
G. Urinary dye studies
H. None of the above

Choose the best management option for each clinical scenario. Each option may be used once, more than once or not at all.

A 34-year-old woman from Zimbabwe presents with continuous dribbling incontinence after having her 2nd child. Apart from prolonged labour the woman denies any complications related to her pregnancies. She is normally fit and well.

A

Vesicovaginal fistulae should be suspected in patients with continuous dribbling incontinence after prolonged labour and from a country with poor obstetric services. A dye stains the urine and hence identifies the presence of a fistula.

129
Q

Theme: Urinary incontinence

A. Bladder diary for 3 days
B. Urodynamic studies
C. Bladder drill training for 6 weeks
D. Pelvic floor exercises 3 months
E. Oxybutynin
F. IV urography
G. Urinary dye studies
H. None of the above

Choose the best management option for each clinical scenario. Each option may be used once, more than once or not at all.

A 53-year-old lady complains of involuntary passage of urine when she coughs or sneezes. She is multiparous G2, P2 with a forceps delivery of her second child.

A

The first line management of stress urinary incontinence, in this case arising for pelvic floor trauma is pelvic floor exercises. These will help a proportion of patients. Non responders should have urodynamics performed to confirm the diagnosis.

130
Q

Theme: Urinary incontinence

A. Bladder diary for 3 days
B. Urodynamic studies
C. Bladder drill training for 6 weeks
D. Pelvic floor exercises 3 months
E. Oxybutynin
F. IV urography
G. Urinary dye studies
H. None of the above

Choose the best management option for each clinical scenario. Each option may be used once, more than once or not at all.

A 56-year-old lady reports incontinence mainly when walking the dog. A bladder diary is inconclusive.

A

Urodynamic studies are indicated when there is diagnostic uncertainty or plans for surgery.

131
Q

A 17-year-old female presents requesting advice as she forgot to take her Microgynon 30 pills on a weekend away. She is normally very good at remembering her pill but has missed days 10, 11 and 12 of her packet and it is now day 13. Although she took the day 13 pill this morning she is concerned she may become pregnant and she had unprotected sexual intercourse whilst away. What is the most appropriate management?

No action needed
No action needed but omit pill break at end of pack
Offer emergency contraception - hormonal
Offer emergency contraception - intrauterine device
No action needed but use condoms for next 7 days
A

Tough question. As the patient had taken the pill for 7 days in a row previously she is protected for the next 7 days. The FSRH guidelines state: ‘after seven consecutive pills have been taken there is no need for emergency contraception’ - please consult the link. The guidelines also recommend in this situation using condoms for the next 7 days

Combined oral contraceptive pill: missed pill

The advice from the Faculty of Sexual and Reproductive Healthcare (FSRH) has changed over recent years. The following recommendations are now made for women taken a combined oral contraceptive (COC) pill containing 30-35 micrograms of ethinylestradiol

If 1 pill is missed (at any time in the cycle)
take the last pill even if it means taking two pills in one day and then continue taking pills daily, one each day
no additional contraceptive protection needed

If 2 or more pills missed
take the last pill even if it means taking two pills in one day, leave any earlier missed pills and then continue taking pills daily, one each day
the women should use condoms or abstain from sex until she has taken pills for 7 days in a row. FSRH: ‘This advice may be overcautious in the second and third weeks, but the advice is a backup in the event that further pills are missed’
if pills are missed in week 1 (Days 1-7): emergency contraception should be considered if she had unprotected sex in the pill-free interval or in week 1
if pills are missed in week 2 (Days 8-14): after seven consecutive days of taking the COC there is no need for emergency contraception*
if pills are missed in week 3 (Days 15-21): she should finish the pills in her current pack and start a new pack the next day; thus omitting the pill free interval

*theoretically women would be protected if they took the COC in a pattern of 7 days on, 7 days off

132
Q

Which one of the following is not a risk factor for the development of pre-eclampsia?

Body mass index of 38 kg/m^2
Smoking
A woman carrying twins
Nulliparity
Diabetes mellitus
A

There is some evidence to suggest that pre-eclampsia is actually less common in smokers

Pre-eclampsia

Pre-eclampsia is a condition seen after 20 weeks gestation characterised by pregnancy-induced hypertension in association with proteinuria (> 0.3g / 24 hours). Oedema used to be third element of the classic triad but is now often not included in the definition as it is not specific

Pre-eclampsia is important as it predisposes to the following problems
fetal: prematurity, intrauterine growth retardation
eclampsia
haemorrhage: placental abruption, intra-abdominal, intra-cerebral
cardiac failure
multi-organ failure

Risk factors
> 40 years old
nulliparity (or new partner)
multiple pregnancy
body mass index > 30 kg/m^2
diabetes mellitus
pregnancy interval of more than 10 years
family history of pre-eclampsia
previous history of pre-eclampsia
pre-existing vascular disease such as hypertension or renal disease

Features of severe pre-eclampsia
hypertension: typically > 170/110 mmHg and proteinuria as above
proteinuria: dipstick ++/+++
headache
visual disturbance
papilloedema
RUQ/epigastric pain
hyperreflexia
platelet count 160/110 mmHg although many clinicians have a lower threshold
oral labetalol is now first-line following the 2010 NICE guidelines. Nifedipine and hydralazine may also be used
delivery of the baby is the most important and definitive management step. The timing depends on the individual clinical scenario

133
Q

A pregnant woman comes to see you for advice. The Down’s syndrome screening tests have indicated she is at high risk and the midwives have discussed the option of an amniocentesis. Which one of the following best describes the timing of an amniocentesis?

Performed at 13-14 weeks, karyotype available after 3 weeks
Performed at 16 weeks, karyotype available after 3 days
Performed at 14-15 weeks, karyotype available after 3 weeks
Performed at 16 weeks, karyotype available after 3 weeks
Performed at 14-15 weeks, karyotype available after 3 days
A

Amniocentesis: performed at 16 weeks, karyotype after 3 weeks

Some laboratories are now able to detect Down’s syndrome using fluorescence in-situ hybridisation (FISH) in 2 days, but full karyotyping takes 3 weeks

Amniocentesis

Amniocentesis is a procedure used in prenatal diagnosis. It may be offered after screening tests have indicated a high risk of fetal abnormality or in women considered to be at high risk, for example if > 35 years old.

Around 20 ml of fluid is removed by transabdominal needle under ultrasound guidance. Fetal cells present in the amniotic fluid are then studied to aid the diagnosis of a number of conditions.

Amniocentesis is usually performed at 16 weeks and the risk of fetal loss is 0.5-1%. The karyotype results typically take 3 weeks. It is known the karyotype may be wrong in 1/1000 cases due to maternal cells being present

Conditions which may be diagnosed
neural tube defects (raised AFP levels in the amniotic fluid)
chromosomal disorders
inborn errors of metabolism

134
Q

A 27-year-old pregnant woman is found to have Chlamydia. What is the most appropriate treatment?

No antibiotic therapy is indicated
Cefixime
Erythromycin
Doxycycline
Ciprofloxacin
A

Erythromycin or amoxicillin is currently recommended for pregnant or breast feeding women. The efficacy of amoxicillin, often assumed to be ineffective against Chlamydia, was supported in a recent Cochrane review. A test of cure should be carried out following treatment

Chlamydia

Chlamydia is the most prevalent sexually transmitted infection in the UK and is caused by Chlamydia trachomatis, an obligate intracellular pathogen. Approximately 1 in 10 young women in the UK have Chlamydia. The incubation period is around 7-21 days, although it should be remembered a large percentage of cases are asymptomatic

Features
asymptomatic in around 70% of women and 50% of men
women: cervicitis (discharge, bleeding), dysuria
men: urethral discharge, dysuria

Potential complications
epididymitis
pelvic inflammatory disease
endometritis
increased incidence of ectopic pregnancies
infertility
reactive arthritis
perihepatitis (Fitz-Hugh-Curtis syndrome)

Investigation
traditional cell culture is no longer widely used
nuclear acid amplification tests (NAATs) are now rapidly emerging as the investigation of choice
urine (first void urine sample), vulvovaginal swab or cervical swab may be tested using the NAAT technique

Screening
in England the National Chlamydia Screening Programme is open to all men and women aged 15-24 years
the 2009 SIGN guidelines support this approach, suggesting screening all sexually active patients aged 15-24 years
relies heavily on opportunistic testing

135
Q

A woman who is 12 weeks pregnant presents as she is concerned following a recent antenatal scan. The scan has reportedly shown increased nuchal translucency. Other than Down’s syndrome, which one of the following is most associated with this finding?

Renal agenesis
Cystic fibrosis
Polyhydramnios
Cytomegalovirus infection
Congenital heart defects
A

Congenital heart defects

Ultrasound in pregnancy

A nuchal scan is performed at 11-13 weeks. Causes of an increased nuchal translucency include:
Down’s syndrome
congenital heart defects
abdominal wall defects

Causes of hyperechogenic bowel:
cystic fibrosis
Down’s syndrome
cytomegalovirus infection

136
Q

A 52-year-old woman presents to surgery seeking advice regarding contraception. She has recently started in a new relationship but is unsure if she requires contraception, as she thinks she may be going through the menopause. She is experiencing hot flushes and her last period was 7 months ago. What is the most appropriate advice?

She no longer requires contraception
Contraception is needed until 12 months after her last period
Contraception is needed until 36 months after her last period
Contraception is needed until 18 months after her last period
Contraception is needed until 24 months after her last period
A

Need for contraception after the menopause
12 months after the last period in women > 50 years
24 months after the last period in women 50 years
24 months after the last period in women

137
Q

Which one of the following statements regarding emergency contraception is true?

The intrauterine device may work by inhibiting fertilisation or implantation
The intrauterine device should be taken out after the next menstrual cycle
The intrauterine device is 95% effective regardless of when it is used during the cycle
Vomiting occurs in around 15% of patients who take emergency hormonal contraception
Emergency hormonal contraception is 95% effective if used between 24-48 hours following intercourse
A

The intrauterine device may work by inhibiting fertilisation or implantation

Emergency contraception

There are two methods currently available in the UK:

Emergency hormonal contraception

There are now two methods of emergency hormonal contraception (‘emergency pill’, ‘morning-after pill’); levonorgestrel and ulipristal, a progesterone receptor modulator.

Levonorgestrel
should be taken as soon as possible - efficacy decreases with time
must be taken within 72 hrs of unprotected sexual intercourse (UPSI)*
single dose of levonorgestrel 1.5mg (a progesterone)
mode of action not fully understood - acts both to stop ovulation and inhibit implantation
84% effective is used within 72 hours of UPSI
levonorgestrel is safe and well tolerated. Disturbance of the current menstrual cycle is seen in a significant minority of women. Vomiting occurs in around 1%
if vomiting occurs within 2 hours then the dose should be repeated
can be used more than once in a menstrual cycle if clinically indicated

Ulipristal
a progesterone receptor modulator currently marketed as EllaOne. The primary mode of action is thought to be inhibition of ovulation
30mg oral dose taken as soon as possible, no later than 120 hours after intercourse
concomitant use with levonorgestrel is not recommended
may reduce the effectiveness of combined oral contraceptive pills and progesterone only pills
caution should be exercised in patients with severe asthma
repeated dosing within the same menstrual cycle is not recommended
breastfeeding should be delayed for one week after taking ulipristal. There are no such restrictions on the use of levonorgestrel

Intrauterine device (IUD)
must be inserted within 5 days of UPSI, or
if a women presents after more than 5 days then an IUD may be fitted up to 5 days after the likely ovulation date
may inhibit fertilisation or implantation
prophylactic antibiotics may be given if the patient is considered to be at high-risk of sexually transmitted infection
is 99% effective regardless of where it is used in the cycle
may be left in-situ to provide long-term contraception. If the client wishes for the IUD to be removed it should be at least kept in until the next period

*may be offered after this period as long as the client is aware of reduced effectiveness and unlicensed indication

138
Q

Which one of the following is most likely to be responsible for menorrhagia?

Pelvic inflammatory disease
Intrauterine system (Mirena)
Hyperthyroidism
Cervical ectropion
Smoking
A

Pelvic inflammatory disease

Menorrhagia: causes

Menorrhagia was previously defined as total blood loss > 80 ml per menses, but it is obviously difficult to quantify. The assessment and management of heavy periods has therefore shifted towards what the woman considers to be excessive and aims to improve quality of life measures.

Causes
dysfunctional uterine bleeding: this describes menorrhagia in the absence of underlying pathology. This accounts for approximately half of patients
anovulatory cycles: these are more common at the extremes of a women’s reproductive life
uterine fibroids
hypothyroidism
intrauterine devices*
pelvic inflammatory disease
bleeding disorders, e.g. von Willebrand disease

*this refers to normal copper coils. Note that the intrauterine system (Mirena) is used to treat menorrhagia

139
Q

Which one of the following is an example of a third generation progestogen?

Levonorgestrel
Drospirenone
Ethynodiol diacetate
Norethisterone
Desogestrel
A

Desogestrel

Progestogen only pill: types

Second generation
norethisterone
levonorgestrel
ethynodiol diacetate

Third generation
desogestrel (Cerazette)

Cerazette
new third generation type of progestogen only pill (POP) containing desogestrel
inhibits ovulation in the majority of women
users can take the pill up to 12 hours late rather than 3 hours like other POPs

140
Q

Antiepileptic medication is known to increase the risk of congenital defects. Which one of the following medications is thought to be associated with the lowest risk?

Levetiracetam
Sodium valproate
Phenytoin
Carbamazepine
Gabapentin
A

Carbamezepine

Epilepsy: pregnancy and breast feeding

The risks of uncontrolled epilepsy during pregnancy generally outweigh the risks of medication to the fetus. All women thinking about becoming pregnant should be advised to take folic acid 5mg per day well before pregnancy to minimise the risk of neural tube defects. Around 1-2% of newborns born to non-epileptic mothers have congenital defects. This rises to 3-4% if the mother takes antiepileptic medication.

Other points
aim for monotherapy
there is no indication to monitor antiepileptic drug levels
sodium valproate: associated with neural tube defects
carbamazepine: often considered the least teratogenic of the older antiepileptics
phenytoin: associated with cleft palate
lamotrigine: studies to date suggest the rate of congenital malformations may be low. The dose of lamotrigine may need to be increased in pregnancy

Breast feeding is generally considered safe for mothers taking antiepileptics with the possible exception of the barbiturates

It is advised that pregnant women taking phenytoin are given vitamin K in the last month of pregnancy to prevent clotting disorders in the newborn

Sodium valproate

The November 2013 issue of the Drug Safety Update also carried a warning about new evidence showing a significant risk of neurodevelopmental delay in children following maternal use of sodium valproate.

The update concludes that sodium valproate should not be used during pregnancy and in women of childbearing age unless clearly necessary. Women of childbearing age should not start treatment without specialist neurological or psychiatric advice.

141
Q

A 58-year-old woman presents for review. Her GP is considering a diagnosis of ovarian cancer as part of the differential diagnosis. Which of the following symptoms is seen in ovarian cancer?

Abdominal swelling
Pelvic pain
Urinary symptoms
Early satiety and bloating
All of the above symptoms may be seen
A

Vague symptoms in an older woman - ?ovarian cancer

There are no symptoms which have a high positive predictive value for ovarian cancer. A high index of suspicion is needed to make the diagnosis

Ovarian cancer

Ovarian cancer is the fifth most common malignancy in females. The peak age of incidence is 60 years and it generally carries a poor prognosis due to late diagnosis. Around 90% of ovarian cancers are epithelial in origin.

Risk factors
family history: mutations of the BRCA1 or the BRCA2 gene
many ovulations: early menarche, late menopause, nulliparity

It is traditionally taught that infertility treatment increases the risk of ovarian cancer, as it increases the number of ovulations. Recent evidence however suggests that there is not a significant link. The combined oral contraceptive pill reduces the risk (fewer ovulations) as does having many pregnancies.

Clinical features are notoriously vague
abdominal distension and bloating
abdominal and pelvic pain
urinary symptoms e.g. Urgency
early satiety
diarrhoea

Diagnosis is difficult and usually involves diagnostic laparotomy

142
Q

Nancy is a 29 year old lady who has given birth to a baby boy 3 days ago and is keen to discuss future contraception. She was previously on the combined pill but is keen to avoid using anything if she can. She is not breast-feeding. How long after giving birth does she not require any contraception?

Up to 28 days
She needs contraception immediately after giving birth
Up to 21 days
Up to 2 months
Up to 14 days
A

Nancy can be informed that she does not require contraception up to 21 days after giving birth.

The following guidance has been taken from the Faculty of Sexual and Reproductive Healthcare Guideline on Postnatal Sexual and Reproductive Health:

Prior to Day 21 postpartum no contraceptive methods are required. In non-breastfeeding
women, ovulation may occur as early as Day 28. As sperm can survive for up to 7 days in the
female genital tract, contraceptive protection is required from Day 21 onwards if pregnancy
is to be avoided.

Women who are breastfeeding and who wish to avoid pregnancy should be advised to use
a contraceptive method. As fertility is reduced, any contraceptive method will be more
effective when used by a breastfeeding woman. Those women who are fully breastfeeding
may wish to rely on the lactational amenorrhoea method (LAM) alone until breastfeeding
reduces or other LAM criteria are no longer fulfilled.

Postnatal Sexual and Reproductive Health - Faculty of Sexual & Reproductive Healthcare Clinical Guidance 2009 - http://www.fsrh.org/pdfs/Ceuguidancepostnatal09.pdf

Post-partum contraception

After giving birth women require contraception after day 21.

Progestogen only pill (POP)
the FSRH advise ‘postpartum women (breastfeeding and non-breastfeeding) can start the POP at any time postpartum.’
after day 21 additional contraception should be used for the first 2 days
a small amount of progestogen enters breast milk but this is not harmful to the infant

Combined oral contraceptive pill (COC)
absolutely contraindicated - UKMEC 4 - if breast feeding

143
Q

What is the main mechanism of action of the combined oral contraceptive pill?

Causes fallopian tube dysfunction
Thickens cervical mucus
Thins endometrial lining
Prevents implantation
Inhibition of ovulation
A

Combined oral contraceptive pill: overview

The combined oral contraceptive pill (COC) is one of the most popular methods of contraception currently used in the UK

Mode of action
main: stops ovulation
also: thickens cervical mucus (reducing chance of semen entering uterus) and thins endometrial lining (reducing chance of implantation)

144
Q

A 27-year-old female presents to her GP as she missed her Cerazette pill (progestogen only) this morning and is unsure what to do. She normally takes the pill at around 0900 and it is now 1430. What advice should be given?

Emergency contraception should be offered
Perform a pregnancy test
Take missed pill as soon as possible and omit pill break at end of pack
Take missed pill now and no further action needed
Take missed pill now and advise condom use until pill taking re-established for 48 hours
A

Take pill now and no further action needed

The missed pill rules for the progestogen only pill (POP) are simpler than those used for the combined oral contraceptive pill, but it is important not to confuse the two.

‘Traditional’ POPs (Micronor, Noriday, Nogeston, Femulen)

If less than 3 hours late
no action required, continue as normal

If more than 3 hours late (i.e. more than 27 hours since the last pill was taken)
action needed - see below

Cerazette (desogestrel)

If less than 12 hours late
no action required, continue as normal

If more than 12 hours late (i.e. more than 36 hours since the last pill was taken)
action needed - see below

Action required, if needed:
take the missed pill as soon as possible. If more than one pill has been missed just take one pill. Take the next pill at the usual time, which may mean taking two pills in one day
continue with rest of pack
extra precautions (e.g. condoms) should be used until pill taking has been re-es

145
Q

A woman rings for advice 18 days post-partum. She is keen to start her progestogen-only pill again. There have been no problems since giving birth and breast feeding is going well. What is the most appropriate advice?

Contraindicated if breast-feeding
Start immediately, effective immediately
Start on day 28, effective after 2 days
Start 2 months post-partum, effective after 2 days
Start 3 months post-partum, effective after 2 days
A

Start immediately, effective immediately

Post-partum contraception

After giving birth women require contraception after day 21.

Progestogen only pill (POP)
the FSRH advise ‘postpartum women (breastfeeding and non-breastfeeding) can start the POP at any time postpartum.’
after day 21 additional contraception should be used for the first 2 days
a small amount of progestogen enters breast milk but this is not harmful to the infant

Combined oral contraceptive pill (COC)
absolutely contraindicated - UKMEC 4 - if breast feeding

146
Q

A 54-year-old woman who has had a hysterectomy presents for advice about hormone replacement therapy. Which one of the following would result from the use of a combined oestrogen-progestogen preparation compared to an oestrogen-only preparation?

Decreased risk of venous thromboembolism
Increased risk of a stroke
Increased risk of breast cancer
Increased risk of endometrial cancer
Better control of symptoms
A

HRT: adding a progestogen increases the risk of breast cancer

This is the rationale behind giving women who’ve had a hysterectomy oestrogen-only treatment. The BNF states that the stroke risk is the same regardless of whether the HRT preparation contains progesterone.

Hormone replacement therapy: adverse effects

Hormone replacement therapy (HRT) involves the use of a small dose of oestrogen (combined with a progestogen in women with a uterus) to help alleviate menopausal symptoms.

Side-effects
nausea
breast tenderness
fluid retention and weight gain

Potential complications
increased risk of breast cancer: increased by the addition of a progestogen
increased risk of endometrial cancer: reduced by the addition of a progestogen but not eliminated completely. The BNF states that the additional risk is eliminated if a progestogen is given continuously
increased risk of venous thromboembolism: increased by the addition of a progestogen
increased risk of stroke
increased risk of ischaemic heart disease if taken more than 10 years after menopause

Breast cancer
in the Women’s Health Initiative (WHI) study there was a relative risk of 1.26 at 5 years of developing breast cancer
the increased risk relates to duration of use
breast cancer incidence is higher in women using combined preparations compared to oestrogen-only preparations
the risk of breast cancer begins to decline when HRT is stopped and by 5 years it reaches the same level as in women who have never taken HRT

147
Q

A pregnant woman asks for advice about alcohol consumption during pregnancy. Which one of the following is in line with current NICE guidelines?

1 to 2 units once or twice per week throughout pregnancy
Avoid first and second trimester. If then chooses to drink 1 to 4 units no more than twice per week
1 to 2 units once per week throughout pregnancy
Avoid first trimester. If then chooses to drink 1 to 2 units once or twice per week
Avoid first trimester. If then chooses to drink less than 7-14 units per week
A

Avoid first trimester. If then chooses to drink 1 to 2 units once or twice per week

Antenatal care: specific points

NICE issued guidelines on routine care for the healthy pregnant woman in March 2008

Nausea and vomiting
natural remedies - ginger and acupuncture on the ‘p6’ point (by the wrist) are recommended by NICE
antihistamines should be used first-line (BNF suggests promethazine as first-line)

Vitamin D
NICE recommend ‘All women should be informed at the booking appointment about the importance for their own and their baby’s health of maintaining adequate vitamin D stores during pregnancy and whilst breastfeeding’
‘women may choose to take 10 micrograms of vitamin D per day, as found in the Healthy Start multivitamin supplement’. This was confirmed in 2012 when the Chief Medical Officer advised: ‘All pregnant and breastfeeding women should take a daily supplement containing 10micrograms of vitamin D, to ensure the mothers requirements for vitamin D are met and to build adequate fetal stores for early infancy’
particular care should be taken with women at risk (e.g. Asian, obese, poor diet)

Alcohol
NICE recommend women should avoid alcohol during the first trimester
if women choose to drink alcohol during pregnancy they should be advised to drink no more than 1 to 2 UK units once or twice a week

148
Q

A woman who is 41 weeks pregnant has a biophysical profile. Which one of the following is not assessed during this test?

Fetal activity
Head circumference
Fetal breathing movements
Fetal tone
Amniotic fluid volume
A

Biophysical profile

A biophysical profile is an antenatal ultrasound test which assesses:
amniotic fluid volume
fetal tone
fetal activity
fetal breathing movements
reactivity of the heart